NOTEPAD
Results
of 200 questions answered correctly

You have reached of 200 points, ( %)

Your time

Question 1 of 200

1. A   30-year-old   patient  complains about   having   abdominal  pain   and   diarrhea for  five  days;  body  temperature rise  up  to  37, 5oC  along  with  chills. The day   before   a   patient  had   been   in   a forest and drunk from an open water reservoir.  Laboratory  analyses   enabled to make the following diagnosis: amebic dysentery. What  is the  drug of choice  for its treatment?

Explanation

Metronidazole forms toxic free radical metabolites in the bacterial cell that damage DNA. It is bactericidal, and an antiprotozoal. It is used to treat Giardia, Entamoeba, Trichomonas, Gardnerella vaginalis, Anaerobes (bacteroides, Clostridium difficile). It can be used with a proton pump inhibitor and clarithromycin for “triple therapy” against Helicobacter pylori.

Entamoeba histolytica causes amebic dysentery.

2.

Quite  often the cause of secondary immunodeficiency is an infection involvement, when the causative agents propagate directly  in the cells of immune system  and  destroy  it. The  following  diseases are characterized by:

Explanation

IMG_9905   

 Secondary immunodeficiency occur after full development of the immune system. Chronic virus infections (e.g. infectious mononucleosis caused by Epstein-Barr virus) and HIV (human immunodeficiency virus) may cause secondary immunodeficiencies. HIV → CD4 cells; Epstein Barr virus (EBV) → B lymphocytes (B cells).

3. Heart rate  of a 30-year-old  man under emotional stress  reached 112 bpm.  The reason   for   the   heart   rate   increase   is the altered condition of the following conducting  system of heart:

Explanation

During stress sympathetic nervous system produces the dominant effects in the body.

The sinoatrial (SA) node is a small strip of modified cardiac muscle, situated in the superior part of lateral wall of right atrium, just below the opening of superior vena cava. The fibers of this node do not have contractile elements. SA node is called the pacemaker because the rate of production of impulse (rhythmicity) is higher in SA node than in other parts. The rate is 60-100/min and higher during tachycardia when stimulated by sympathetic effects. Atrioventricular (AV) node is 40-60/min.

4.

Sanitary  bacteriological research on water by the membrane filter method revealed  two red colonies on a membrane filter (Endo agar)  through which 500 ml of analyzed  water  were passed.  Calculate the coli index and coli titer of the analyzed water:

Explanation

1.     4 and 250

Coli titer is the smallest amount of water where 1 E.coli is present.

Coli index is the amount of E. coli in 1 liter of water.

            2 E.coli   -  500ml

            1 E.coli   -  ?

                        ? = 250

Therefore, Coli titer = 250

 

1 liter of water  =  1000ml

            2 E. coli   -  500ml

                   ?       -   1000ml (1L)

                        ? = 4

Therefore, Coli index = 4

 

Coliform index (Coli index) and Coli titer are used to rate the purity of water, soil and air based on the count of fecal bacteria by testing for coliforms especially the well known Escherichia coli (E. coli).

5. A   46-year-old    female    patient   has a       continuous   history    of    progressive muscular   (Duchenne’s)  dystrophy.   Which blood  enzyme  changes  will be  of  diagnostic value in this case?

Explanation

Creatine kinase/Creatine phosphokinase is an enzyme present in striated muscles, used to generate energy. When its serum level is elevated it is often an indication of muscle damage caused by injury, muscular dystrophy or cardiac problems. Most forms of muscular dystrophy are associated with decreased creatinine excretion. Creatinine is a break down product of creatine. Normal constituent of urine but can be elevated in muscular dystrophy. 

6.

Extensive    thromboembolic   infarction   of   the   left   cerebral  hemispheres, large septic spleen, immunocomplex glomerulonephritis, ulcers on the edges of the  aortic  valves, covered  with polypous thrombus with colonies of staphylococcus were  revealed   on  autopsy  of  the  young man  who  died  in  coma.  What   disease caused cerebral thromboemboli?

Explanation

1.     Septic (bacterial) endocarditis is the form of sepsis, for which septic lesion of valves of the heart is characteristic. The most often causative agents are staphylococcus albus, staphylococcus aureus, streptococcus viridian and enterococcus. Polypous-ulcerative endocarditis develops on both sclerotic and intact valves. Large thromboembolic polyp-shaped plaques appear on sclerotic valves. The spleen is enlarged due to prolonged pulp hyperplasia (“septic spleen”). Immune-complex diffuse glomerulonephritis develops in the kidneys, foci of softening and hemorrhages are observed in the brain due to vascular changes (vasculitis, aneurysm) and thromboembolism.

7.

A  10-year-old   girl  has  a  history   of repeated acute respiratory viral infection. After  recovering  she presents with multiple petechial hemorrhages on the sites of friction  from  clothing  rubbing   the  skin. What kind of hypovitaminosis has this girl?

Explanation

Vitamin C (ascorbic acid): found in fruits and vegetables; an antioxidant; also facilitates iron absorption by reducing it to Fe2+ state. It is necessary for hydroxylation of proline and lysine in collagen synthesis; necessary for dopamine β-hydroxylase, which converts dopamine to norepinephrine. Deficiency leads to: scurvy – swollen gums, bruising, petechiae, hemarthrosis, anemia, poor wound healing, perifollicular and subperiosteal hemorrhages, “corkscrew” hair; Weakened immune response.

Type III collagen is found in blood vessels; Type IV collagen is found in basement membrane. Deficiency in Vitamin C disrupts the second stage of collagen synthesis in fibroblasts (hydroxylation of collagen) which results in petechiae, bruising, hemarthrosis.

Vitamin B2 (riboflavin) deficiency – growth retardation, glossitis, conjunctivitis

Vitamin B1 (thiamine) deficiency – Beri-Beri (polyneuritis)

Vitamin A (retinol) deficiency – Night blindness

 

8. Autopsy  of  a  patient  who  suffered from croupous  pneumonia and died from pneumococcal sepsis  revealed  900 ml of turbid   greenish-yellow  liquid  in  the  right pleural  cavity. Pleural  leaves are dull, plephoric. Name the clinicopathological form of inflammation in the pleural cavity:

Explanation

     Complications of croupous or lobar pneumonia includes empyema, gangrene, carnification, abscess formation, bacteremic spread which leads to purulent meningitis, bacterial endocarditis, arthritis, pericarditis and other organs. Empyema is a purulent inflammation of serous membranes (empyema of pleura, empyema of gall bladder and urinary bladder etc). Purulent or suppurative inflammation is characterized by the production of large amounts of pus or purulent exudates (liquids) consisting of a lot of neutrophils, necrotic cells and edema fluid.

9.

Researchers   isolated    5   isoenzymic forms  of lactate  dehydrogenase from  the human blood serum and studied their properties. What  property indicates  that the  isoenzymic  forms  were  isolated  from the same enzyme?

Explanation

       Isoenzyme is any of several forms of an enzyme that all catalyze the same reaction but may differ in reaction rate, inhibition by various substances, electrophoretic mobility or immunologic properties. Examples of important enzymes with isoenzymic forms are:

·        Lactate dehydrogenase: LDH1, LDH2, LDH3, LDH4, LDH5, (5 isoenzymic forms).

 

·        Creatine kinase (CK): CK-MM; CK-MB; CK-BB (3 isoenzymic forms)

10. A patient suffering from stomach ulcer has  been   treated with  an  antacid   drug almagel. For acute bronchitis treatment he was prescribed the  antibiotic methacycline.   However    within   next   5  days   the fever   didn’t   fall,   cough    and    sputum nature  remained unchanged. A  physician came to the  conclusion  that  the  drugs were incompatible. What type of drug incompatibility is the case?

Explanation

       Pharmacokinetics refers to what the body does to a drug. Once a drug is administered through one of several available routes, 4 pharmacokinetic properties determine the speed of onset of drug action, intensity of the drug’s effect and the duration of drug action: absorption, distribution, metabolism and elimination.

Absorption: first, drug absorption from the site of administration permits entry of the therapeutic agent (either directly or indirectly) into plasma. Absorption is the transfer of a drug from its site of administration to the bloodstream via different mechanisms.

Methacycline is a tetracycline. All tetracyclines are adequately absorbed after oral ingestion. However, taking these drugs concomitantly with diary foods in the diet decreases absorption due to the formation of nonabsorbable chelates of the tetracyclines with calcium ions. Nonabsorbable chelates are also formed with other divalent and trivalent cations e.g. those found in magnesium and aluminum antacids and in iron preparations. This presents a problem if a patient self-treats the epigastric upsets caused by tetracycline ingestion with antacids.

 

Nonforming CO2 antacids: aluminum hydroxide + magnesium hydroxide = Almagel or Maalox.

11.

ECG  of a 44-year-old  patient shows signs  of  hypertrophy  of  both   ventricles and the right atrium.  The  patient was diagnosed  with the tricuspid  valve insufficiency. What pathogenetic variant of cardiac dysfunction is usually observed in case of such insufficiency?

Explanation

1.     Due to incomplete closure of the tricuspid valve during right ventricular systole, part of blood is regurgitated into the right atrium, where it is mixed with the normal volume of blood delivered from the venae cavae which makes the atrium become distended and hypertrophied. During diastole, a larger volume of blood is delivered into the right ventricle because the portion of blood that was regurgitated into the atrium during systole is added to the normal volume of blood delivered → ↑volume (heart overload). This causes dilatation and hypertrophy of the right ventricle. Compensation in this disease is attained by intensified work of left ventricle which leads to its hypertrophy too. The mass of the circulating blood usually increase proportionally to the degree of circulatory insufficiency. This is favored by retention of sodium chloride and water in decreased renal filtration and increased reabsorption of sodium and the increasing number of RBCs (hypoxia is attended by intensified hemopoiesis to compensate for the developing insufficiency) → ↑blood volume.

12.

Proserin increases skeletal muscle tone when given systematically. Halothane induces   relaxation  of  skeletal   muscles and   reduces   proserin  effects.   What   is the nature of proserin and halothane interaction?

Explanation

     Proserin (Neostigmine) is an indirect acting cholinergic agonist (anticholinesterase). It preserves endogenous acetylcholine which can stimulate a greater number of acetylcholine receptors at the muscle endplate. Halothane, on the other hand, is a general anesthetic that breaks neurotransmission. Proserin stimulates and aids neurotransmission, while halothane breaks neurotransmission (antagonism). This is done indirectly because they don’t act by the same mechanism to antagonize each other. It is functional because each drug weakens the other’s action/effect.

13. The minute  blood volume in a patient with transplanted heart  has increased  as a result  of physical  activity.  What  regulative mechanism  is responsible for these changes?

Explanation

1.     A transplanted heart is denervated, so no nervous stimulation can increase or decrease the heart rate or cardiac output. Therefore, the only mechanism capable of increasing minute blood volume is the catecholamines (epinephrine/adrenaline; norepinephrine/noradrenaline).

14.

An aged man had raise of arterial pressure under  a stress. It was caused  by activation of:

Explanation

     Stress activates the sympathoadrenal system and the hypothalamic-pituitary-adrenocortical (HPA) axis. Defense reactions involve catecholamine release, vagal withdrawal, cortisol secretion and activation of the rennin-angiotensin system. Catecholamine release is capable of increasing the arterial pressure. Sympathoadrenal system involves the sympathetic nervous system and the adrenal glands especially increased sympathetic activity that causes increased secretion of epinephrine by the adrenal medulla and norepinephrine by the postganglionic sympathetic nerve endings.

15. Autopsy  of  a  man  who  died  from chronic  cardiovascular collapse  revealed \\\"tiger  heart\\\". Sidewards  of endocardium a yellowish-white banding can be seen; myocardium is dull, dark-yellow.  What process caused this pathology?

Explanation

1.     Intracellular accumulations (parenchymal degeneration or dystrophies). Intracellular fatty degenerations are the abnormal accumulations of triglycerides within parenchymal cells. The liver, heart, kidneys are damaged the most frequently. Fatty degeneration of the heart – Tiger’s heart; fatty degeneration of the liver – Goose liver; fatty degeneration of the kidney – large white kidney.

16. A 55-year-old  male patient was hospitalised  to  a  surgical  clinic for  suspected septicemia.    What    material   should    be taken  for analysis?

Explanation

     Septicemia is a clinical form of sepsis that indicates bacteria in the blood. In cases of septicemia, blood is inoculated into glucose broth (sugar broth) and the isolated pure culture is tested for its hemolytic properties. Glucose is a sugar that some bacteria can use because of an enzyme that begins the breakdown of this compound.

17.

A  patient has  pellagra.  Interrogation revealed  that he had lived mostly on maize for a long time  and  eaten  little  meat. This disease had been caused by the deficit of the following substance  in the maize:

Explanation

Vit. B3 (Niacin, PP): coenzyme forms NAD+, NADP+ is derived from tryptophan. Synthesis requires Vit. B2 and B6Vitamin B3 (niacin) is very high in fish and meat products.

18.

During    examination  of   a   6-year- old    child    a    doctor     revealed     greyish    films    on    the    pharyngeal    tonsils. Their removal  provoked moderate haemorrhage. Bacterioscopy revealed gram-positive clublike bacteria. What symptoms will develop in this child within the next few days if no specific treatment is provided?

Explanation

There are two types of fibrinous inflammation: croupous and diphtheric fibrinous inflammation. Usually croupous inflammation develops on the columnar epithelium. In this case the fibrinous membranes unfix easily, without any effort. Diphtheric fibrinous inflammation develops on the squamous or intermediate epithelium and the fibrinous membranes unfix with difficulties and may even bleed when trying to unfix it.

“their removal provoked moderate hemorrhage” – Diphtheric inflammation. General changes in diphtheria are accompanied by toxinemia and appear as toxic lesions to myocardium, nerves (peripheral nervous system) adrenal glands, kidney (interstitial nephritis) and liver.

19. A 46-year old patient consulted a doctor complaining about joint pain that becomes stronger the day before the weather changes. Blood examination revealed   an  increased   concentration  of uric acid. The most probable cause of the disease is the intensified  disintegration of the following substance:

Explanation

image

krushkrok No21 (2014)

The end product of the purine nucleotides catabolism in humans and other primates is uric acid (urate) which is excreted in urine. Allopurinol and febuxostat inhibits Xanthine oxidase (XO). Hypoxanthine and Xanthine which is more soluble is excreted in urine. Purine nucleotides (adenine and guanine).  AMP – Adenosine monophosphate; GMP – Guanosine monophosphate

Pyrimidine (thymine, Uracil, cytosine); catabolism of thymine and uracil gives urea, while catabolism of cytosine gives β-alanine.

20. 12 hours after an acute attack  of retrosternal  pain  a  patient presented a jump of aspartate aminotransferase activity in blood  serum. What pathology is this deviation  typical for?

Explanation

IMG_9910

In laboratory diagnosis of acute myocardial infarction: Creatine kinase isoenzyme MB (CK-MB) appears within 4-8hrs, peaks at 24hrs and disappears within 1.5-3 days. Sensitivity and specificity is 95%. Reappearance of CK-MB after 3days – reinfarction. CK-MM – found in skeletal muscle and heart (not specific for heart). LDH1 – heart muscle; LDH2 – blood serum. LDH levels are also high in tissue breakdown or hemolysis. Although CK-MB is more specific and sensitive for infarction than LDH. Aspartate transaminase is not specific for heart damage alone but can still be used to diagnose myocardial infarction. But troponin test [cardiac troponins I (cTnI) and T (cTnT)] is the most sensitive and specific test for myocardial infarction.

21.

After   a  trauma of  the  upper   third of the  anterior forearm surface  a patient presents with difficult pronation, weakening of palmar  flexor  muscles  and  altered skin sensitivity of 1-3 fingers. Which nerve is damaged?

Explanation

krushkrok No29 (2013)

krushkrok No29a (2013)

Median nerve arises from the brachial plexus & receives contributions from both the medial & lateral cords. It gives NO branch in the arm area (i.e from origin to cubital fossa). It supplies both pronators (pronator teres &  quadratus), and all flexors except the flexor carpi ulnaris & the ulnar half of the flexor digitorum profundus. In the palmar area, the cutaneous branches supply the skin of the palmar surface of fingers 1-3 including the radial aspect of skin of the palmar surface of fingers 1-3 including the radial aspect of the 4th finger.

22.

A  2-year-old  child  with  mental   and physical  retardation has  been  delivered to  a hospital.  He  presents with frequent vomiting after having meals. There is phenylpyruvic acid in urine. Which metabolism abnormality is the reason  for this pathology?

Explanation

1.     Phenylketones – phenylacetate, phenyllactate and phenylpyruvate. These compounds are not normally produced in significant amounts in the presence of functional phenylalanine hydroxylase, but are elevated in phenylketonuria. These metabolites give urine a characteristic musty (“mousey”) odour. The disease acquired its name from the presence of a phenylketone (now known to be phenylpyruvate) in the urine.

 

Phenylketonuria (PKU) is caused by a deficiency of phenylalanine hydroxylase. PKU is the most common clinically encountered inborn error of amino-acid metabolism. Biochemically, it is characterized by accumulation of phenylalanine and a deficiency of tyrosine. It may also be caused by deficiencies in tetrahydrobiopterin cofactor (BH4) or in dihydropteridine (BH2) reductase, which regenerates BH4 from BH2.

23. A 38-year-old man died in the attempt of lifting weight. He had collaptoid state. Autopsy revealed  an  extensive  aneurism rupture  of  thoracic   aorta.   He   suffered from   visceral  syphilis  during   his  lifetime. What pathological process caused weakness  of aortic  wall, its dilatation and rupture?

Explanation

     image

image

The main forms of syphilis are Gumma; syphilitic aortitis; neurological syphilis. In syphilitic aortitis, the aorta is affected by an infiltration of lymphocytes and plasma cells beginning around the vasa vasorum and extending into the media, causing weakening due to focal destruction (windowing) of the specialized elastic tissues. There is compensatory irregular thickening of the intima (tree-bark appearance), but the important effect is expanding aneurysm formation because the elastic fibers are destroyed.

24.

A  patient suffering  from  myasthenia has been  administered proserin. After  its administration the patient has got nausea, diarrhea, twitch  of  tongue   and  skeletal muscles. What  drug  would  help to eliminate the intoxication?

Explanation

1.     Neostigmine/proserin reversibly inhibits anticholinesterase, an indirect acting cholinergic agonists. It is used symptomatically to treat myasthenia gravis. Proserin does not cause CNS side effects and is not used to overcome toxicity of central acting antimuscarinic agents.

Atropine is an antimuscarinic agent and is used for the treatment of overdoses or intoxication of cholinesterase inhibitor (proserin, physostigmine) and some types of mushroom poisoning (certain mushrooms contain cholinergic substances that block cholinesterases). It also blocks the effects of excess acetylcholine resulting from acetylcholinesterase inhibitors.

Physostigmine and pyridostigmine bromide are acetylcholinesterase inhibitors and will further synergize proserine.

 

Mesaton is an α-adrenergic agonists (adrenomimetics); Isadrin is a β-adrenergic agonists.

25.

A  man  died  from  an  acute  infectious disease accompanied by fever, jaundice,    haemorrhagic   rash    on    the    skin and  mucous   membranes  as  well  as  by acute renal insufficiency. Histological examination of  renal  tissue  (stained by Romanovsky-Giemsa method) revealed some   convoluted  bacteria  looking   like C und S letters.  What bacteria were revealed?

Explanation

krushkrok No25 (2011)

     Three genera of spirochetes cause human infection:

·        Treponema: causes syphilis and the nonveneral treponematoses. Morphology – thin, tight spirals

·        Borrelia: causes lyme disease and relapsing fever. Morphology – large and loosely coiled

·        Leptospira: causes leptospirosis. Morphology – thin, tight spirals

Leptospira is tightly coiled, fine spirochetes that are not stained with dyes but are seen by darkfield microscopy. Leptospira interrogans is the cause of leptospirosis. Human infection results when leptospira is ingested or pass through mucus membranes or skin. They circulate in the blood and multiply in various organs, producing fever and dysfunction of the liver (jaundice), kidney (uremia), lungs (hemorrhage) and CNS (aseptic meningitis). Leptospira C and S letters.

26.

A  46-year-old  patient suffering  from the diffuse toxic goiter underwent resection   of   the    thyroid    gland.   After    the surgery the patient presents with appetite loss, dyspepsia,  increased neuromuscular excitement. The body weight remained unchanged. Body  temperature is normal. Which of the following has caused such a condition in this patient?

Explanation

image One of the most common complications of thyroidectomy (resection of thyroid gland) is hypoparathyroidism because the parathyroid gland lying behind the thyroid gland capsule can be accidentally removed during the surgery by unskilled surgeons. Accidental removal of the parathyroid gland results in hypoparathyroidism.
27. Medical   examination  at   the   military registration and enlistment office revealed   that  a  15-year-old  boy  was  high, with eunuchoid body proportions, gynecomastia,  female   pattern  of  pubic hair distribution. The boy had also fat deposits  on the  thighs,  no facial hair,  high voice, subnormal intelligence  quotient. Which karyotype corresponds with this disease?

Explanation

krushkrok No70 (2007)

Barr body is an inactive X-chromosome. So a boy (XY) with an inactive X-chromosome must have an additional X-chromosome – XXY (Klinefelter’s syndrome). Causes :

* nondisjunction (maternal and paternal nondisjunction in meiosis I)

* Mosaicism: with the karyotype being 46, XY/47, XXY

Manifestations: gynecomastia, female pattern of pubic hair distribution, no facial hair, high voice.

FullSizeRender (1)

28.

A  doctor  recommends a  patient with  duodenal ulcer  to  drink  cabbage  and potato  juice   after   the   therapy  course. Which substances  contained in these vegetables  help  to  heal  and  prevent the ulcers?

Explanation

     Cabbage juice is one of the most healing nutrients for ulcer repair as it is a huge source of vitamin U (vitamin U is actually not a vitamin but an enzyme known as S-methylmethionine). Research shows that vitamin U, administered as raw cabbage juice, is effective in promoting the rapid healing of peptic ulcers. Potatoes on the other hand are rich sources of vitamins and minerals. Several studies, such as the one published in the June 2008 edition of the “Journal of vascular nursing”, have shown reduced levels of vitamin A in patients suffering from leg ulcers, indicating a connection between the two, probably because vitamin A helps to form and maintain healthy skin, mucous membrane and teeth. Vitamin A and the amino acid glutamine help to regenerate healthy epithelial cells.

29. Urine  analysis  of  a  12-year-old  boy reveals high concentration of all aliphatic amino  acids  with  the  highest  excretion of cystine and cysteine. US of kidneys revealed  kidney concrements. What is the most likely pathology?

Explanation

     Cystinuria is a disorder of the proximal tubules reabsorption of filtered cystine and dibasic amino acids (lysine, ornithine, arginine). The inability to reabsorb cystine leads to accumulation and subsequent precipitation of stones of cystine in the urinary tract. It is an inherited disorder of amino acid transport. The disease expresses itself clinically by the precipitation of cystine to form kidney stones (calculi), which can block the urinary tract. Oral hydration is an important part of treatment for this disorder. Cystine is formed from two molecules of cysteine joined together.

30.

A 5-year-old child has been diagnosed with    acute     right    distal    pneumonia. Sputum  inoculation revealed  that the causative  agent  is resistant  to  penicillin, but it is sensitive to macrolides.  What drug should be prescribed?

Explanation

     Macrolides inhibit protein synthesis by blocking translocation, they bind to the 23S rRNA of the 50S ribosomal subunit. It is bacteriostatic. Examples of macrolides includes azithromycin, clarithromycin, erythromycin etc.

All other options are not macrolides, but are antibiotics.

Ampicillin is an extended spectrum penicillin; a β-lactam antibiotic.

 

Gentamycin and streptomycin are Aminoglycosides. Together with tetracyclines, they are 30S ribosomal subunit inhibitor.

31. Autopsy of a 73-year-old man who had been suffering from the coronary heart disease  along  with cardiac  insufficiency for a long time revealed:  nutmeg  liver, brown induration of  lungs,  cyanotic  induration of kidneys  and  spleen.  What  kind  of circulation  disorder was the  cause  of such effects?

Explanation

FullSizeRender (30)

In general, these are signs of both right-sided cardiac insufficiency (nutmeg liver – venous congestion) and left-sided cardiac insufficiency (brown induration of lungs – venous congestion). It is Chronic because the patient has been suffering for a long time. In venous or passive hyperemia or congestion, morphologically, the sectioned surface of lungs is dark brown and the process is named brown induration of the lungs. Spleen – cyanotic induration of the spleen; liver – nutmeg liver.

  Acute heart failure refers to sudden and rapid onset of signs and symptoms of abnormal heart functions. Chronic heart failure is characterized by the symptoms that appear slowly over a period of time and become worst gradually.

The main symptoms of right-sided heart failure are fluid accumulation and swelling (edema) in the feet, ankles, legs, liver and abdomen. Left-sided heart failure leads to fluid accumulation in the lungs, which causes shortness of breath. At first, shortness of breath occurs only during exertion, but as heart failure progresses, it occurs with less and less exertion and eventually occurs even at rest. Moist and dry rales are heard over the lungs.

32. A patient suffering from chronic hyperacidic gastritis takes an antacid drug  for heartburn elimination. After  its ingestion the patient feels better but at the same time he has a sensation  of stomach swelling. Which of the following drugs might be the cause of such side effect?

Explanation

     Antacids are weak bases that react with gastric acid to form water and a salt to diminish gastric acidity. Because pepsin is inactive at a pH greater than 4, antacids also reduce pepsin activity. They can be:

·        CO2 forming: sodium hydrocarbonate (or carbonate); calcium carbonate

·        Non-forming CO2: magnesium hydroxide, aluminum hydroxide

Uses: hyperacid gastritis, gastric and duodenal ulcers; poisoning with acids.

 

Adverse effects: in addition to the potential for systemic alkalosis, sodium hydrocarbonate liberates CO2 causing belching and flatulence (sensation of stomach swelling); aluminum hydroxide tends to cause constipation, whereas magnesium hydroxide tends to produce diarrhea.

33.

A 30-year-old  male patient with acute pancreatitis has been found to have a disorder   of   cavitary   protein  digestion. The reason  for such condition can be the hyposynthesis  and  hyposecretion of  the following enzyme:

Explanation

Acute pancreatitis → ↓production of pancreatic juice.

 

Trypsin is the most powerful proteolytic enzyme in pancreatic juice. Pepsin is secreted by chief cells in the stomach.

Acute pancreatitis is an autodigestion of pancreas by pancreatic enzymes (proteolytic enzymes). Contrycal (Aprotinin, Gordox)- protease inhibitor. Therefore,it can inhibit the proteolytic enzymes digesting the pancreas. 

34. A patient has been given high doses of hydrocortisone for a long time. This caused   atrophy of one   of  the   adrenal cortex zones. Which zone is it?

Explanation

FullSizeRender (22)

The adrenal gland (suprarenal gland) has a secretory parenchymal tissue organized into cortical and medullary regions. Adrenal cortex (derived from mesoderm; steroid-secreting portion) and medulla (derived from neural crest; catecholamine-secreting portion). The adrenal cortex is divided into 3 zones on the basis of arrangement of its cells:

* Zona glomerulosa (15%): arranged in closely packed ovoid clusters; secretes aldosterone.

* Zona fasciculata (80%): large and polyhedral; arranged in long straight cords; secretes cortisol.

* Zona reticularis (5-7%): cells are arranged in anastomosing cords separated by fenestrated capillaries; secretes androgens.

Hydrocortisone is a glucocorticoid (cortisol) and an intake of hydrocortisone will decrease endogenous production of cortisol from zona fasciculata of adrenal cortex leading to its atrophy.

35. A victim of an accident  has bleeding from the  soft tissues  anteriad the  mandibular  angle.  Which  vessel  should  be  ligated for the bleeding arrest?

Explanation

krushkrok No6 (2012)

The facial artery (arteria facialis) arises somewhat above the lingual artery from the external carotid artery. Then the artery enters the submandibular gland to give the glandular branches. Upon passing the gland, the artery loops around the mandible edge (inferior border), anterior to the mandibular angle to reach the facial area. It ascends along the anterior border of the masseter muscle in direction of the medial angle of eye. At the medial angle of eye, it gives its terminal branch – angular artery which forms an anastomosis with dorsal nasal artery of ophthalmic artery, thereby creating an anastomosis which connects both internal and external carotid arteries.

36.

Microscopic  examination of a  gram-stained scrape from a patient’s tongue revealed  oval, round,  elongated chains of dark-violet gemmating cells. What disease can be caused by this causative agent?

Explanation

     There are two types of fungi: yeasts and molds. Yeasts grow as single cells that reproduce by asexual budding (gemmating). Molds grow as long filaments (hyphae) and form a mat (mycelium). Candida albicans is an oval yeast with a single bud. It is part of the normal flora of mouth (oral cavity), mucous membranes of upper respiratory, gastrointestinal and female genital tracts. In tissues it may appear as yeast or as pseudohyphae. Pseudohyphae are elongated yeasts (elongated chains) that visually resemble hyphae but are not true hyphae. Candida albicans causes thrush, vaginitis, esophagitis and chronic mucocutaneous candidiasis.

37. The greater amount of nitrogen is excreted from  the  organism  in  form  of urea. Inhibition of urea synthesis and accumulation of  ammonia in  blood  and tissues are induced by the decreased activity of the following liver enzyme:

Explanation

image       

Urea is the major disposal form of amino groups derived from amino acids and account for about 90% of the nitrogen-containing components of urine. Formation of carbomoyl phosphate by carbomoyl phosphate synthetase I is driven by cleavage of two molecules of ATP. Ammonia is incorporated into carbamoyl phosphate. Ultimately, the nitrogen atom derived from this ammonia becomes one of the nitrogens of urea. NB: carbamoyl phosphate synthetase II participates in the biosynthesis of pyrimidines. However, when the function of carbomoyl phosphate synthetase I is compromised, blood ammonia levels rise and urea synthesis is disturbed.

38. A 36-year-old female patient has a history of collagen disease. Urine analysis is likely to reveal an increased concentration of the following metabolite:

Explanation

Collagen, most abundant protein in human body; organizes and strengthens extracellular matrix. Collagen contains Gly-X-Y (X and Y are proline or lysine). Glycine(Gly) makes 1/3 of collagen. Oxyproline (hydroxyproline) is a major collagen amino acid which enables it to be regarded as a marker that reflects the catabolism of collagen. 

39.

A coprological  survey revealed  light-colored  feces containing drops  of neutral fat. The most likely reason  for this condition is the disorder of:

Explanation

Obturation (obstruction, to close) of bile duct – it can be:

* Intrahepatic – blockage of intrahepatic bile ducts

* Extrahepatic – blockage of common bile duct (ductus choledochus).

Findings:

* malabsorption: bile salts do not enter the Small Intestine; no emulsification of fat.

*light coloured stool: due to lack of urobilin (which leads to lack of stercobilin).

*Jaundice (posthepatic, mechanic, obstructive): increased conjugated Bilirubin.

* Steatorrhea

 

The findings are specific for obstruction of bile duct and bile acid deficiency.

40. The  secretion  of  which  hypophysial hormones will be  inhibited after  taking the  oral contraceptives containing sex hormones?

Explanation

     Follicle stimulating hormone and the luteinizing hormone are together called gonadotropic hormones or gonadotropins because of their action on gonads. Products containing a combination of an estrogen and a progestin are the most common type of oral contraceptives. The estrogen provides a negative feedback on the release of luteinizing hormone (LH) and follicle stimulating hormone (FSH) by the pituitary gland, thus preventing ovulation. The progestin also inhibits LH release and thickens the cervical mucus, thus hampering the transport of sperm. Oral contraceptives are the drugs taken orally (by mouth) to prevent pregnancy.

41.

A histological specimen of a kidney shows a part of the distal tubule going between the  afferent and  efferent arteriole. The cells building the tubule wall have dense nuclei; basal membrane is absent. Such structural formation is called:

Explanation

  image

 krushkrok No140 (2012)

Juxtaglomerular apparatus is formed by 3 different structures:

·        Macula densa: is the end portion of thick ascending segment as it opens into the distal convoluted tubule. It is situated between the afferent and efferent arterioles of the same nephron. It is very close to afferent arteriole. Macula densa is formed by tightly packed cuboidal epithelial cells.

·        Mesangial cells: are situated in the triangular region bound by afferent arteriole, efferent arteriole and macula densa.

·        Juxtaglomerular cells: are specialized smooth muscle cells situated in the wall of afferent arteriole just before it enters the Bowman’s capsule.

42.

During preparation of a patient to a heart surgery it was necessary  to measure pressure  in  heart   chambers.  In  one   of them  pressure varied  from  0 mm Hg up to  120 mm  Hg  within  one  cardiac  cycle. What heart chamber is it?

Explanation

       Pressure in each heart chambers:

 

    Diastole (mmHg)

      Systole (mmHg)

Left ventricle

               4-12

            90-140

Right ventricle

               0-8

             15-28

Left atrium

          -2 to +6

               6-20

Right atrium

          -2 to +6

               2-14

 

Only the left ventricle can range from 0-120mmHg

43.

A  patient with  essential  hypertension  has  a high  rate  of blood  renin.  Which of antihypertensive drugs should be preferred?

Explanation

image

Lisinopril is an angiotensin converting enzyme inhibitor (ACE Inhibitor). This drug block the ACE that cleaves angiotensin I to form the potent vasoconstrictor angiotensin II. Rennin converts angiotensinogen to angiotensin I.

Propranolol is a β-blocker; Prazosin – α1-adrenoblocker; Nifedipine – Ca2+-channel blocker; Dichlothiazide – thiazide diuretic.

For a high rate of renin, the antihypertensive drugs that are effective are: renin inhibitor (Aliskiren); angiotensin II receptor blocker and angiotensin converting enzyme (ACE) inhibitors because they block the renin –angiotensin system.

44.

In order  to determine toxigenicity  of diphtheria bacilli  a  strip  of  filter  paper impregnated with antitoxic  diphtherial serum was put on the dense nutrient medium.  There   were  also  inoculated  a  microbial  culture  under  examination and  a strain  that  is known  to  be  toxigenic.  If the   microbial   culture   under   examination  produces exotoxin,  this  will result  in formation of:

Explanation

     The presence of the antitoxic diphtheria serum (antibody) and the exotoxin (antigen) cause an interaction between them leading to the production of antigen-antibody complexes producing precipitates (precipitin lines) on the strip of filter paper. We will have a precipitin ring if a tube were to be used instead of a filter paper.

45.

It  was  revealed   that   T-lymphocytes were  affected   by  HIV.  Virus  enzyme  - reverse  transcriptase   (RNA-dependent DNA-polymerase)  -  catalyzes the synthesis of:

Explanation

     Human immunodeficiency virus (HIV): diploid genome (2 molecules of RNA). The 3 structural genes(i.e. proteins coded for by the genes) are:

·        env (gp 120 and gp 41): formed from cleavage of gp 160 to form envelope glycoproteins. gp 120 is for attachment to host CD4+ T cell. gp 41 is for fusion and entry.

·        Gag (p24): capsid protein

·        pol: reverse transcriptase, aspartate protease, integrase.

ELISA/Western blot (immunoblot) tests look for antibodies to the viral proteins listed above.

Reverse transcriptase synthesizes dsDNA (ds-double stranded) from genomic RNA (mRNA); dsDNA integrates into host genome. Virus binds CD4 as well as a coreceptor, either CCR5 on macrophages (early infection) or CXCR4 on I cells (late infection).

·        Homozygous CCR5 mutation – immunity

 

·        Heterozygous CCR5 mutation – slower course.

46.

Microscopical examination of an enlarged cervical lymph node revealed blurring  of its structure, absence  of lymphoid  follicles; all the  microscopic  fields showed cells with roundish nuclei and thin limbus of basophil cytoplasm. It is known from the clinical data that other groups  of lymph  nodes  are  also enlarged as well as spleen  and  liver. What disease might be suspected?

Explanation

        Lymphoid leukosis is a type of malignant neoplasm characterized by the proliferation of cells native to the lymphoid tissues; that is - lymphocytes, histiocytes and their precursors and derivatives. It is characterized by an enlarged liver due to infiltration of cancerous lymphoid cells. In addition, other abdominal organs are also affected.

47.

A 38-year-old  patient came to a traumatology   centre    and    complained about  an injury of his right hand. Objectively: the  patient has a cut  wound  in the region of the thenar eminence on the right hand; distal phalanx  of the I finger cannot be flexed. What muscle was injured?

Explanation

     The muscles of the hand are divided into 3 groups: muscles of the thumb (thenar muscles); muscle of the fifth digit (hypothenar eminence) and central palmar muscles. Flexor pollicis longus (long flexor muscle of thumb) resides laterally in the deep layer. It flexes the thumb.  Its tendon passes under the flexor retinaculum onto the palm and inserts into the base of the distal phalanx of the thumb.

Flexor pollicis brevis (short flexor muscle of thumb) and abductor pollicis brevis (short abductor muscle of thumb) attaches to the base of the proximal phalanx of the thumb. Opponens pollicis (opposer muscle of thumb) attaches to the first metacarpal bone. It is the deepest of the thenar muscles. Abductor pollicis (abductor muscle of thumb) inserts into the base of the proximal phalanx of the thumb.

 

NB: first (I) finger – Thumb

48.

Lung   ventilation   in   a   person    is increased   as  a  result  of  physical  activity. Which  of the  following  indices  of the external  respiration is much  higher  than in a state of rest?

Explanation

        During physical activity all lung volumes tend to increase to meet up with the increased demand. Therefore, the whole respiratory volume is increased – inspiratory reserve volume, expiratory reserve volume, tidal volume; vital capacity includes inspiratory reserve volume, tidal volume and expiratory reserve volume. Total lung capacity is the volume of air present in lungs after a deep (maximal) inspiration.

49.

As  a  result  of  activation   of  the  ion channels  of  the  external membrane the rest   potential  of  an  excitable   cell  has greatly increased. What channels were activated?

Explanation

     Increase in rest potential of an excitable cell is hyperpolarization. Hyperpolarization is often caused by efflux of K+ (a cation) through K+-channels or influx of Cl- (an anion) through Cl- channels. Hyperpolarization is the opposite of depolarization. Influx of Na+ or Ca2+ causes depolarization. Depolarization decreases rest potential.

50.

As a result   of  continuous  starvation the glomerular filtration  rate has increased   by  20%.  The   most   probable cause of the glomerular filtration alteration under the mentioned conditions is:

Explanation

     Starvation decreases protein synthesis because the substrates are not readily available. Decrease in protein synthesis decreases oncotic pressure and thereby increasing glomerular filtration. Net filtration pressure is the balance between hydrostatic pressure (glomerular capillary pressure) and oncotic pressure.

Net filtration pressure = Hydrostatic pressure – Oncotic pressure

 

Therefore, a decrease in oncotic pressure increases the filtration rate according to the degree of starvation which decreases protein synthesis.

51.

A patient with cardiogenic  shock, hypotension, asphyxia  and  edemata was given an injection  of non-glycosidic cardiotonic. What drug was injected?

Explanation

     

 Non-glycoside cardiotonics includes β1-adrenergic agonists, phosphodiesterase inhibitors, these drugs prolong synthesis of ATP and have positive inotropic property and increase cardiac output.

Dobutamine – β1-adrenergic agonists

Caffeine sodium benzoate; Cordiamin; Aethimizolum and Bemegride – Analeptics.

52.

After  transfusion of 200 ml of blood a patient presented with body temperature rise up  to 37, 9oC . Which  of the  following substances  is the  most  likely cause  of temperature rise?

Explanation

     The transfused blood might contain pyrogens or its an incompatible blood that activates the immune system. Pyrogenic cytokines, principally IL-1; IL-6 and TNF-α are released into the bloodstream for transport to the hypothalamus, where they exert their action. These cytokines induce prostaglandin E2 (PGE2), which is a metabolite of arachidonic acid. It is hypothesized that when IL- 1β interacts with the endothelial cells of the blood-brain barrier in the capillaries of the organum vasculosum laminae terminalis (OVLT), which is in the third ventricle above the optic chiasm, PGE2 is released into the hypothalamus. At this point PGE2 binds to receptors in the hypothalamus to induce increase in the thermostatic set point through the second messenger cAMP. In response to the increase in its thermostatic set point, the hypothalamus initiates shivering and vasoconstriction that raise the body’s core temperature to the new set point and fever is established.

53.

A man who is riding the carousel presents with increased  heart  rate, sweating, nausea.  This condition is caused  primarily by the stimulation of the following receptors:

Explanation

image     

The receptor areas of the vestibular labyrinth are represented with the following structures:

·        Macula of Utricle

·        Macula of Saccule

·        Ampullary crests

The first two detect linear motion, while the ampullary crest detect angular motion. The vestibulocochlear nerve supply these receptors. Both maculae consist of the sensory hair cells covered with jelly-like susbstance. The substance contains the crystals of calcium carbonate called otoliths (vestibular otolith). The utricle and saccule detect linear movement, also contributing to balance. The ampullary crests reside within each membranous ampulla. They also comprise the sensory hair cells covered with the same jelly-like substance called the ampullary cupula. The ampullae is the sensory organ in the semicircular canal that sense angular (rotational) acceleration of the head, thereby regulating balance. NB: carousel is a revolving belt.

54.

A worker  of a cattle  farm fell acutely ill and then died from the progressing intoxication. Autopsy revealed  enlarged, hyposthenic spleen  of dark-cherry colour when  dissected;  excessive  pulp  scraping. At the base and fornix of brain pia maters are edematous, soaked  with blood,  dark-red  (\\\"scarlet hat\\\").   Microscopic   examination revealed serous haemorrhagic inflammation  of  brain  tissues  and  tunics along  with destruction of small vessel walls. What is the most likely diagnosis?

Explanation

There are two medically important Bacillus species: Bacillus anthracis and Bacillus cereus. Bacillus anthracis causes anthrax. Human disease occurs in 3 main forms: cutaneous, pulmonary (inhalation) and gastrointestinal. Humans are most often infected cutaneously at the time of trauma to the skin, which allows the spores on animal products such as hides, bristles and wool to enter. Spores can also be inhaled or when contaminated meat is ingested. After being inhaled, the organism moves rapidly to the mediastinal lymph nodes and causes hemorrhagic mediastinitis. Pathogenesis is based on the production of two exotoxins  (Anthrax toxin) – edema factor and lethal factor. Hemorrhagic mediastinitis, septic shock hemorrhagic meningitis and death are severe life-threatening complications. In fatal cases, the organism may affect the spleen, liver, intestines, kidneys, adrenal glands and meninges.

55. Which of the listed diuretic agents WILL NOT have diuretic  effect on a patient with Addison’s disease?

Explanation

IMG_9896      

Addison’s disease is chronic adrenal insufficiency (hypoaldosteronism). Spironolactone is an aldosterone antagonist. In addison’s disease, there is no aldosterone or it is very low; therefore, spironolactone will not have any effect on such patient.

56. A man with a wound of his limb that had been suppurating for a long time died from intoxication.  Autopsy revealed extreme emaciation, dehydration, brown atrophy of liver, myocardium, spleen  and cross-striated muscles as well as renal amyloidosis. What diagnosis corresponds with the described  presentations?

Explanation

     Chronic sepsis (chroniosepsis): this form of sepsis is characterized by durably availability, not healing of primary septic focus that progress into a chronic one. In organs and tissues, there is atrophy and dehydration are expressed. Brown atrophy is found in the liver, myocardium and striated muscles. The spleen is decreased.

57.

This drug  has  a destructive effect  on erythrocytic forms  of malarial  plasmodia and  dysenteric amoebae.  It  is  used  for treatment and prevention of such diseases as malaria,  amebiasis  and  interstitial disease. What drug is it?

Explanation

image Chingamin (chloroquine) prevents polymerization of the hemoglobin breakdown product (heme) into hemozoin. Intracellular accumulation of heme is toxic to the parasite. It is a weak base and may buffer intracellular pH, thereby inhibiting cellular invasion by parasitic organisms. The drug is solely a blood schizonticide (erythrocytic) and will not eradicate secondary tissue schizonts. It is the drug of choice for acute attacks, prophylaxis and treatment of malaria, has been used in amebic liver disease in combination with metronidazole and in autoimmune disorders including rheumatoid arthritis, systemic lupus erythematous (interstitial disease).
58.

A  patient has  an  increased pyruvate concentration  in  blood,   most   of   it  is excreted with the urine. What kind of avitaminosis  has this patient?

Explanation

       Thiamine (vitamin B1): thiamine pyrophosphate (TPP) is the biologically active form of the vitamin, formed by the transfer of a pyrophosphate group from ATP to thiamine. Biological role of TPP: it is a component of pyruvate dehydrogenase and α-ketoglutarate dehydrogenase complexes catalyzing the reactions of oxidative decarboxylation of pyruvate and α-ketoglutarate (kreb’s cycle) i.e. it promotes energy formation from carbohydrates and lipids. It’s also a component of transketolase (pentose phosphate pathway of glucose oxidation) essential for fats and nucleic acids synthesis.

Pyruvate to acetyl CoA + CO2 reaction

 

            If pyruvate dehydrogenase cannot function properly due to vitamin B1 deficiency, then pyruvate will be accumulated in blood because it can’t be broken down.

59. A child suffers from drug idiosyncrasy. What is the cause of such reaction?

Explanation

Idiosyncrasy and allergic reaction is as a result of organism’s increased sensitivity to the drugs. It’s a complication of pharmacological therapy (adverse reactions). 

* Idiosyncrasy is genetically conditioned perverse organism’s response to drugs (e.g. inefficiency, lack of enzymes participating in drug metabolism). Genetic deficiency of glucose 6-phosphate dehydrogenase (hereditary enzymopathy). This enzyme is needed for proper functioning of glutathione (an antioxidant) to prevent the oxidative effect of antimalarial drugs on the RBCs.

60.

One  of  the  parents is  suspected   of having phenylketonuria recessive gene. What  is the  risk of giving birth  to a child with inborn  phenylketonuria?

Explanation

        Only one of the parents has a recessive gene

AA      Aa (crossing) = All the children are healthy (0%)

Phenylketonuria is a recessive genetic defect and it needs to be homozygous recessive (aa) to be expressed.

61.

Pulmonary  examination  of  a  patient  who  has  worked  as  a  stone  grinder for 9 years  revealed  small dense  roundish  nodules   consisting   of  connective  tissue. The nodules were found to have peripheral macrophages. Such pulmonary  alterations are indicative  of the following disease:

Explanation

image       

Silicosis is a lung disease that is caused by inhaling tiny bits of silica. Silica is a common mineral that is part of sand, rock and mineral ores like quartz. People who work in jobs where they could breathe in these tiny silica bits like sandblasting, mining, construction etc; are at risk for silicosis. The silica dust can cause fluid buildup and scar tissue in the lungs that cuts down the ability to breathe. Silicosis cannot be cured, but you can prevent it if you take specific steps to protect yourself.

62.

A 12-year-old teenager has significantly put off weight within 3 months; glucose concentration rose  up  to  50 millimole/l. He fell into a coma. What is the main mechanism of its development?

Explanation

        Hyperosmolar coma is clinically defined by the presence of relative insulin deficiency and hyperglycemia (high glucose concentration), usually higher than 33.3mmol/L with associated elevated serum osmolality (>300mosm/kg), dehydration and stupor, progressing to coma if uncorrected, without the presence of ketosis or acidosis.

Hypoglycemic coma → ↓ glucose concentration

Ketonemic coma → ↑ ketone bodies

 

Lactacidemic coma → ↑ lactic acid

63.

Which  way  of  heat  emission  by  the bodies  of greenhouse workers  is the most effective   at   the   temperature  of   36oC degrees and relative humidity of 70%?

Explanation

Evaporation is a way the body dissipates heat to the environment by its evaporation via sweat or evaporation of moisture from the skin and respiratory tract mucous membranes of (“wet” heat loss). Evaporation closely related to relative humidity.

Heat Radiation is a way the surface of the human body emits heat to the environment in the form of infrared rays. The amount of heat the body radiates to the environment is proportional to the surface of radiation area and to the difference between the mean values of skin and environment temperature. The surface radiation area is the total surface area of body parts that contact the air. Elimination of heat by radiation increases with a decrease in ambient temperature and decreases with its increase. It is possible to reduce elimination of heat by radiation via reduction of the surface of radiation area (“winding oneself into a ball”). Heat radiation does not require a medium for transfer of heat. (Key words: naked or lightly clothed).

Convection is a way the body eliminates heat by means of transferring heat via moving particles of air or water. To dissipate heat by means of convection, body surface shall be airflowed at a temperature that is lower than the temperature of the skin. At that, air layer contacting with the skin warms up, decreases its density, rises and is replaced by cooler, denser air. By increasing the speed of the air flow (wind, ventilation) heat emission increases significantly as well (forced convection). Convection requires convection current; current of gases or liquids (Key words: air over exposed area of skin).

 

Conduction is a way the body eliminates heat by means of direct contact with another object. Heat is transferred down the temperature gradient (i.e. from the object of higher temperature to the object of lower temperature). Conduction requires contact with another object (Key words:  in water). 

64.

Examination  of  a  70-year-old   patient  revealed  insulin-dependent diabetes. What drug should be administered?

Explanation

Sulfonylurea are insulin secretagogues because they promote insulin release from the β-cells of the pancreas. The primary drugs used today are the 2nd generation drugs – glibenclamide, glipizide, glimepiride etc. These agents are useful in the treatment of patients who have type 2 diabetes mellitus and cannot be managed by diet alone. Metformin is the only currently available biguanide; its classed as an insulin sensitizer. Acarbose – α glucosidase inhibitor. Butamide (tolbutamide) is also a sulfonylurea but is older and glibenclamide is more potent and used much more commonly. Actrapid (soluble insulin) is fast acting but not a sulfonylurea.

65.

A disaster  fighter  at a nuclear  power plant  developed  hemorrhagic syndrome on the  background of acute  radiation disease. What  is the  most  important factor of syndrome  pathogenesis?

Explanation

     Radiation disease, radiation therapy, chemotherapy or toxic chemicals can destroy megakryocytes. Megakaryocytes are responsible for the production of thrombocytes (platelets) which can lead to thrombocytopenia. Thrombocytopenia is an abnormally low level of platelets in blood, as a result of reduced platelet production in the bone marrow or excessive peripheral destruction of platelets. People with severe thrombocytopenia may have abnormal bleeding (hemorrhagic syndrome) almost anywhere in the body.

66.

Jaundice treatment involves administration of barbiturates inducing the synthesis  of UDP-glucuronyl transferase. A medicinal  effect is caused by the production of:

Explanation

 

Indirect; Hemolytic; Prehepatic

Mixed; Parenchymal; Hepatic

Direct; Obstructive; Mechanic; Posthepatic

Stercobilin (faeces)

        ↑↑↑

Decreases (pale faces)

Absent (clay coloured faeces)

Type of bilirubin in blood

Unconjugated

Conjugated and Unconjugated

Conjugated

Crigler Najjar syndrome type II is a rare hereditary disorder and is due to a less severe defect in the bilirubin conjugation. It is believed that hepatic UDP-glucuronyltransferase that catalyzes the addition of second glucuronyl group is defective. It is treated with Phenobarbital ( a barbiturate), because it induces the action of UDP-glucuronyltransferase, thereby producing more conjugated (direct) bilirubin.

67. A   histological    specimen    shows   a blood  vessel. Its  inner  coat  is composed by    endothelium,   subendothelium   and internal  elastic  membrane.  The   middle coat  is enriched with  smooth  myocytes. Such morphological characteristics are typical for the following vessel:

Explanation

The tunics of veins are not as distinct or well defined as the tunics of arteries. Veins are divided into 3 types:

* Small veins/venules: postcapillary and muscular venules

* Medium veins

* Large veins

Arteries:

·        Large or elastic arteries

·        Medium or muscular arteries

·        Small arteries and arterioles

Muscular venules are distinguished from postcapillary venules by the presence of a tunica media. Postcapillary venules possess an endothelial lining with its basal lamina and pericytes. Postcapillary venules have no true tunica media.

·        Tunica intima: consists of endothelium with its basal lamina

·        Tunica media: smooth muscle cells

·        Tunica adventitia: collagen fibers

krushkrok No67 (2011)

A vessel without tunica media, also lack muscular tissue. Muscular artery, arteriole and artery of mixed type all have tunica media. Only capillaries and postcapillary venules lack tunica media.

Also a prominent internal elastic membrane helps to distinguish muscular arteries from elastic arteries and muscular venules.

krushkrok No67a (2011)

68.

An  18-year-old  man  was delivered  to the hospital  after a road accident.  Examination  at the traumatological department revealed multiple injuries of soft tissues of face in the region of the medial eye angle. The injuries caused massive haemorrhage. What   arterial  anastomosis  might   have been damaged in this region?

Explanation

krushkrok No68a (2011)   krushkrok No6 (2012)

 Common carotid artery branches to give: external carotid artery and internal carotid artery.

External carotid artery → facial artery → angular artery

Internal carotid artery → ophthalmic artery → dorsal nasal artery

Angular artery is the terminal segment of facial artery which branches within the medial angle of the eye. It anastomoses with the branches of the ophthalmic artery (i.e. the dorsal nasal artery). The two big arteries connected are the external carotid artery and the internal carotid artery.

69.

Autopsy of a 75-year-old  man  with a long history  of atherosclerosis revealed  a grey irregular-shaped focus of loose consistency in the right parietotemporal region of brain.  What  is the most likely cause of this process?

Explanation

The middle (medial) cerebral artery arises from the internal carotid artery and enters the lateral sulcus. There, it gives branches that supply the greater portion of the superolateral surface of the cerebral hemisphere; including the frontal, parietal, temporal lobes and the insula.

Anterior cerebral artery supplies the medial surface of hemisphere.

Posterior cerebral artery supplies the inferior surface of temporal lobe and the inferior and medial surfaces of the occipital lobe

Basilar artery gives the following branches: posterior cerebral arteries, anterior inferior cerebellar artery, superior cerebellar artery, pontine arteries.

 

            Only the middle (medial) cerebral artery supplies the right and left parietotemporal region of brain.

70.

Autopsy  of   a   man   with   a   malignant    stomach    tumour   who   had   died   from   cancer    intoxication  revealed in the posteroinferior lung fields some dense, grayish-red  irregular foci protruding above the section surface. Microscopic examination  revealed   exudate  containing a large  amount of neutrophils in the lumen and walls of small bronchi and alveoles. Such pulmonary alterations indicate the following disease:

Explanation

        Bronchopneumonia (focal pneumonia) is marked by patchy exudative consolidation of lung parenchyma. Bronchopneumonia often is a complication of other disease (complication of stomach cancer). Initially bronchi are affected. Then, inflammation spreads to parenchyma of lungs with accumulation of exudates in the alveoli. Grossly, the lungs show dispersed, elevated, focal areas of palpable consolidation and suppuration. Histological features consist of acute (neutrophilic) suppurative or mixed exudates filling airspaces and airways, usually around bronchi and bronchioles.

71. A  surgeon  has  to  find  the  common hepatic duct during the operative intervention on account of concrements in the  gall ducts.  The  common  hepatic  duct is located  between the leaves of:

Explanation

krushkrok No95a (2012)

FullSizeRender (35)      

 Hepatoduodenal ligament running from the porta hepatica to the superior part of the duodenum. It consists of double layer of peritoneum and enfolds 3 structures:

·        Common hepatic artery

·        Portal vein

·        Common bile duct

Portal vein is localized posteriorly; hepatic artery – left anterior and bile duct – right anterior.

72.

Autopsy   of    a    1,5-year-old    child revealed  haemorrhagic skin rash, moderate hyperaemia and edema  of nasopharyngeal mucous membrane, small haemorrhages in the mucous  membranes and  internal organs;  dramatic  dystrophic alterations in liver and myocardium; acute necrotic nephrosis; massive haemorrhages in the  adrenal glands.  What  disease  are these alterations the most typical for?

Explanation

Morphologically, meningitis can be: meningococcal nasopharyngitis, meningococcal meningitis, meningococcemia. In meningococcemia, changes on the organs are characterized by generalized damage of microcirculation, skin rash, changes in the joints, vascular membrane of the eyes, adrenal glands and kidneys. Changes in the serous layers of the pericardium are observed. The rash is hemorrhagic, star-like, located mainly on the buttocks, lower extremities, eyelids and scleras. Focal necrosis and hemorrhages or bilateral massive hemorrhages with the development of acute adrenal insufficiency (waterhouse-friderichsen syndrome) are noted in the adrenals. Necrosis of nephrothelium of the tubules (necrotic nephrosis) is observed in the kidneys.

Meningococcal meningitis is characterized by hyperemia of pia mater (extremely plethoric), by the end of the 2nd – 3rd day the exudate becomes thicker, green-yellow, purulent (yellow-green cap). Dull-serous exudates during the first days of the disease.

73.

A   75-year-old-female patient   with complaints of visual impairment has been delivered to the ophthalmologic department. Objective examination revealed a brain tumor in area of the left optic tract.  The patient has a visual field defect in the following area:

Explanation

image

Lesions involving the optic tract (i.e. beyond the optic chiasm) produce loss of vision within similar areas of the visual field in both eyes (“homonymous hemianopsia”) – left optic tract – left half of both eyes. These lesions can be caused by stroke, tumor, vascular malformations, demyelinating lesions and abscesses.

74. A   patient  with   coronary    disease has been diagnosed with myocardial hypertrophy, tachycardia and  a decrease in  minute   blood   volume.   What   is  the leading  mechanism  of cardiac  histiocyte damage in this case?

Explanation

     Coronary heart disease leads to ischemia of myocardium. Ischemia → ↓O2 → ↓mitochondrial O2 supply → ↓ATP synthesis → ↓Na+K+ ATPase pump action → ↑Intracellular Na+, Ca2+ → ↑Ca2+ load of mitochondria → uncoupling of oxidative phosphorylation → Metabolic derangement → Cell injury; also ↑Intracellular Na+, Ca2+ → ↑H2O → Acute swelling of cell → enlargement of endoplasmic reticulum → ↓protein synthesis → Metabolic derangement → Cell injury.

75. Blood    analysis   of   a   patient   with jaundice reveals conjugated bilirubinemia, increased concentration of bile acids. There  is no  stercobilinogen in urine. What type of jaundice  is it?

Explanation

Obturation (obstruction, to close) of bile duct – it can be:

* Intrahepatic – blockage of intrahepatic bile ducts

* Extrahepatic – blockage of common bile duct (ductus choledochus).

Findings:

* malabsorption: bile salts do not enter the Small Intestine; no emulsification of fat.

*light coloured stool: due to lack of urobilin (which leads to lack of stercobilin).

*Jaundice (posthepatic, mechanic, obstructive): increased conjugated Bilirubin.

* Steatorrhea

 

The findings are specific for obstruction of bile duct and bile acid deficiency.

 

Indirect; Hemolytic; Prehepatic

Mixed; Parenchymal; Hepatic

Direct; Obstructive; Mechanic; Posthepatic

Stercobilin (faeces)

        ↑↑↑

Decreases (pale faces)

Absent (clay coloured faeces)

Type of bilirubin in blood

Unconjugated

Conjugated and Unconjugated

Conjugated

76. While  studying  maximally  spiralized chromosomes of human karyotype the process of cell division was stopped in the following phase:

Explanation

krushkrok No170 (2013)

Gametogenesis involves the generation of germ cells (gametes) through meiosis. Meiosis requires 2 cell divisions, meiosis I and II, in order to reduce the number of chromosomes to the haploid number of 23. At the beginning of meiosis I, germ cells replicate their DNA so that each of the 46 chromosomes is duplicated into sister chromatids. Homologous pairs then separate into two daughter cells, thereby reducing the chromosome number from diploid to haploid. Shortly thereafter, meiosis II separates sister chromatids (2 sister chromatids are joined in the middle by a centromere). Each gamete then conatins 23 chromosomes.

Metaphase is a stage of cell division in which chromosomes are at their most condensed and coiled stage and chromosomes are arranged along the equatorial plane during this phase. Cell division is usually studied during this phase.

77.

A man is being measured power inputs on  an  empty  stomach,  in the  lying position, under  conditions of physical and psychic rest at a comfortable temperature. Power inputs will reach the maximum at:

Explanation

1.     5-6pm

78.

When  measuring   power  inputs  of  a man  by  the  method  of  indirect   calorimetry the following results were obtained: 1000 ml oxygen consumption and 800 ml carbon  dioxide liberation per minute.  The man under  examination has the following respiratory coefficient:

Explanation

Minute O2 uptake – 1000ml

    Minute CO2 emission – 800ml

    Respiratory Quotient (RQ) = vCO2 /vO2 = 800/1000(ml) = 0.8

 

   

79.

A newborn child with pylorostenosis has often  repeating vomiting  accompanied by apathy, weakness, hypertonicity, sometimes    convulsions.   What   disorder form of acid-base  balance  is it?

Explanation

Acidosis is the reduction in pH (increase in H+ concentration) below normal range. pH is less than 7.35; it is produced by:

·        Increase in partial pressure of CO2 in the body.

·        Decrease in HCO3- concentration.

Alkalosis is the increase in pH (decrease in H+ concentration) above normal range. pH is greater than 7.45; it is produced by:

·        Decrease in partial pressure of CO2 in the body.

·        Increase in HCO3- concentration.

Each of these two disorders has respiratory and non-respiratory forms. The non-respiratory form is divided into metabolic and excretory(non-gaseous).

·        Respiratory acidosis is the acidosis that is caused by alveolar hypoventilation e.g. airway obstruction due to bronchitis or lung diseases (pneumonia).

·        Respiratory alkalosis is caused by alveolar hyperventilation e.g. hypoxia in high altitude.

·        Non-respiratory:

-Metabolic acidosis is characterized by excess accumulation of organic acids such as lactic acid, ketoacids and uric acid formed by normal metabolism e.g. in Diabetes mellitus or extreme/prolonged exercise.

-Excretory/Non-gaseous acidosis may develop in impaired renal H+ excretion related to increased loss of bicarbonate in urine; diarrhea causes acidosis by the loss of bicarbonate with faeces.

-Excretory/Non-gaseous alkalosis: vomiting (loss of gastric acid), increased metabolism of lactate and citrate (turns into bicarbonate and water), long-term use of thiazides and loop diuretics.

 

      It is excretory/non-gaseous alkalosis because of the frequent vomiting.

80.

A concentrated solution of sodium chloride was intravenously injected to an animal. This caused decreased reabsorption of sodium ions in the renal tubules.  It is the result of the following changes  of hormonal secretion:

Explanation

Aldosterone produced in adrenal cortex (zona glomerulosa): causes increased sodium (Na+) reabsorption; increased potassium and hydrogen ions (↑K+, H+) excretion. They increase sodium (↑Na+) channel and Na+ /K+-pump insertion in principal cells of collecting duct; enhances K+ and H+ excretion by way of principal cell K+ channels and α-intercalated cell H+ ATPases of collecting duct. Therefore, increase in aldosterone → ↑ K+ in urine (excretion) and ↓ Na+ in urine (↑ reabsorption); And decreased aldosterone → ↓ K+ excretion (↓K+ in urine) and ↓ Na+  reabsorption (i.e. ↑Na+ in urine); same effects on sweats glands too.

81. A  patient has  been  diagnosed   with alkaptonuria. Choose  an  enzyme  whose deficiency can be the reason for this pathology:

Explanation

image

Ochronosis (Alkaptonuria): congenital deficiency of homogentisate oxidase (homogentisic acid oxidase) in the degradative pathway of tyrosine to Fumarate → pigment-forming homogentisic acid accumulates (homogentisuria) in tissues. Autosomal recessive. Usually benign. Urine turns black on prolonged exposure to air. May have debilitating arthralgias (homogentisic acid toxic to cartilage).

82. A patient consulted an urologist about pain during urination. Analysis of his urine taken  in the daytime  revealed  eggs with a characteristic sharp point. It is known from  the  anamnesis that  the  patient has recently  returned from Australia. What is the most likely diagnosis?

Explanation

The most important trematodes (flukes) are Schitosoma species (blood flukes). Schistosoma causes Schistosomiasis. Schistosoma mansoni and Schistosoma japonicum affect the GIT, whereas Schistosoma haematobium affects the urinary tract. Schistosoma haematobium’s eggs have a terminal spine (sharp point) and lives in the veins draining the urinary bladder. Schistosoma haematobium is found in Africa and the middle-east. Diagnosis depends on finding the characteristic ova (egg) in faeces or urine.

83.

A 49-year old female patient has limitation  of left limbs arbitrary movements. Muscular tonus of left hand and leg is overstrained and spasmodic,  local tendon reflexes  are  strong,  pathological reflexes are  presented. What   is  the  most  likely development  mechanism   of  hypertension and hyperreflexia?

Explanation

     Spasticity is probably caused by the removal of inhibitory influences exerted by the cortex (UMN) on the postural centers of the vestibular nuclei and reticular formation. Lesions in the upper motor neuron (UMN) causes hypertonia and spasticity paralysis, increased reflexes (in UMN – everything ↑). Paralysis is the complete loss of strength and functions of muscle group or a limb.

84.

A 35-year-old patient complains about having  severe  rhinitis  and  loss  of  sense of smell for a week. Objectively: the nasal cavity contains a lot of mucus covering the mucous membrane and blocking olfactory receptors. In what region of the nasal cavity are these receptors located?

Explanation

     Olfactory receptors are situated in olfactory mucus membrane, which is the modified mucus membrane that lines upper part of nostril (superior nasal concha). Olfactory receptor cell is a bipolar neuron. Dendrite of this neuron is short and it has an expanded end called olfactory rod. From the olfactory rod, about 10-12 cilia arise. These cilia project to the surface of olfactory mucus membrane in the upper part of nostril. Mucus secreted by Bowman glands continuously lines the olfactory mucosa.

85.

A 10-year-old  child had  the  mantoux tuberculin  test   administered.  48  hours later  a  papule  up  to  8 mm  in  diameter appeared  on  the   site  of  the   injection. What type of hypersensitivity reaction developed after the tuberculin injection?

Explanation

FullSizeRender (7)

The Mantoux skin test should be read between 48 and 72hrs after administration. The basis of reading  is the presence or absence of induration, which may be determined by inspection and by palpation. A record should also be made of formation of vesicles, bullae, lymphangitis, ulceration and necrosis at the test site. The formation of vesicles, bullae or necrosis at the test site indicates positive result. A negative mantoux result usually signifies that the individual has never been exposed to Mycobacterium tuberculosis i.e. absence of cell mediated immunity to tuberculin.

Mantoux test is a type IV Hypersensitivity reaction (HSR), which involves macrophages,T-lymphocytes and lymphokines(cytokines). Mononuclear cells (lymphocytes,monocytes,macrophages).

Remember,it is antibody independent (i.e does not involve antibodies).

B-lymphocytesàPlasma cellsàIg(Antibodies)------- none is involved in Type IV HSR.
86.

Vitamin B1  deficiency causes disturbance   of   oxidative    decarboxylation  of  α-ketoglutaric acid.  This  leads  to the  impaired synthesis  of  the  following coenzyme:

Explanation

       Thiamine (vitamin B1): thiamine pyrophosphate (TPP) is the biologically active form of the vitamin, formed by the transfer of a pyrophosphate group from ATP to thiamine. Biological role of TPP: it is a component of pyruvate dehydrogenase and α-ketoglutarate dehydrogenase complexes catalyzing the reactions of oxidative decarboxylation of pyruvate and α-ketoglutarate (kreb’s cycle) i.e. it promotes energy formation from carbohydrates and lipids. It’s also a component of transketolase (pentose phosphate pathway of glucose oxidation) essential for fats and nucleic acids synthesis.

Pyruvate to acetyl CoA + CO2 reaction

 

            If pyruvate dehydrogenase cannot function properly due to vitamin B1 deficiency, then pyruvate will be accumulated in blood because it can’t be broken down.

87.

Examination of a pregnant woman having   Rh-negative  blood   revealed   high  level  of  antierythrocytic  antibodies. For  its reduction she  was implanted with  her  husband’s  Rh-positive skin  graft. The   graft   was  rejected  in  two   weeks. Its  microscopic  examination revealed  circulatory disturbance, edema  and cellular infiltration  with lymphocytes,  neutrophils and macrophages predominance. What  is the most likely pathology?

Explanation

     Reaction of transplant rejection resembles delayed hypersensitivity reaction. Transplant antigens induce the production of antibodies and sensibilized lymphocytes, which infiltrate the transplant. Microscopically, lymphohistiocytic infiltration is observed in the transplant. Cellular infiltration causes the disturbance of blood circulation and edema; as a result degenerations and necrosis of transplant develop. The neutrophils and macrophages appear in the transplant. Enzyme destruction of the transplant begins, which is followed by its rejection. Transplant – Graft.

 

This is an immune reaction against the transplant - GRAFT IMMUNITY.

88. A  17-year-old  girl  took  a  high  dose of   phenobarbital   to   commit    a   suicide. An ambulance doctor  cleansed her stomach and gave her an intravenous injection of bemegride and sodium hydrocarbonate solution.  What  was sodium hydrocarbonate injected for?

Explanation

       Sodium hydrocarbonate/sodium bicarbonate is a systemic alkalinizing agent which when given intravenously will increase plasma bicarbonate, buffer excess hydrogen ion concentration, raise blood pH and reverse the clinical manifestations of acidosis. Sodium bicarbonate is used to increase urinary pH in order to increase the solubility of certain weak acids (e.g cystine, sulphonamides, uric acid) and in the treatment of certain intoxications (e.g. methanol, Phenobarbital, salicylates) to decrease renal reabsorption of the drug or to correct acidosis.

89.

While examining  the oral cavity a stomatologist revealed  inflammation of papillae   on  the   border  of  the   median and posterior third of the back of tongue. What papillae are inflamed?

Explanation

krushkrok No89 (2011)

Lingual papillae cover the dorsal surface of the tongue anterior to the sulcus terminalis of the tongue. 4 types:

Circumvallate papillae: large, dome-shaped structures that reside in the mucosa just anterior to the sulcus terminalis . It divides the tongue into anterior 2/3 and posterior 1/3.  Human tongue has 8-12 of it. It has taste buds.

* Filiform papillae: smallest and most numerous. No taste buds, serve only a mechanical role, distributed over the entire anterior dorsal surface. They are the ones that will be coated since they are numerous and serve mechanical role.

* Fungiform papillae: mushroom shaped, have taste buds

* Foliate papillae: occur on the lateral edge of the tongue.

Taste buds are present on fungiform, foliate and circumvallate papillae.

90.

A patient is 44 years old. Laboratory examination of  his  blood  revealed   that content of proteins  in plasma  was 40 g/l. What   influence   will  be  exerted on  the transcapillary water metabolism?

Explanation

        Reference interval (adults): 60 – 80g/L

40g/L is low i.e. a low oncotic pressure.

Glomerular filtration rate (GFR) is inversely proportional to oncotic pressure/colloid osmotic pressure, which is exerted by plasma proteins in the glomerular capillary blood. When oncotic pressure is low, GFR increases. Reabsorption is decreased, because the low oncotic pressure is not sufficient to reabsorb water and other substances (cannot create enough pressure to reabsorb water and other substances).

91. A  patient has  corestenoma. What  is the reason of such condition?

Explanation

Corestenoma is narrowing of the pupil i.e. miosis. Miosis (myosis) is a parasympathetic effect mediated by acetylcholine.

Muscarinic receptors belong to the class of G protein-coupled receptors. These receptors, in addition to binding Acetylcholine, it also recognizes muscarine – an alkaloid that is present in certain poisonous mushrooms. Muscarinic effects include miosis, spasm of accommodation, increasing of lacrimal, salivary, gastrointestinal, bronchial and sweat glands secretion, increase of smooth muscle tonus of internal organs, bradycardia, decreased arterial pressure. 

92.

A  46-year-old   man   had   a  bulging dark macula on skin that caused no discomfort.  With  time  it began  to increase in size and became  painful. It turned dark brown and there  was a nodule  on palpation. Histological  examination of tissues revealed spindle and polymorphous cells with multiple mitoses. Their cytoplasm contained brown  pigment.  What  tumour is it?

Explanation

Melanoma is strongly linked to acquired mutations caused by exposure to ultraviolet (UV) radiation in sunlight. Melanomas show striking variations in colour, appearing in shades of black, brown, red, dark blue and gray. The borders of melanomas are irregular and often notched, unlike the smooth, round and uniform borders of melanocytic nevi. The positive DOPA reaction indicates the presence of melanocytes. Melanoma arises in the skin, meninges, uvea of the eye.

93.

The    patient   with    complaints   of permanent thirst  applied   to  the  doctor. Hyperglycemia, polyuria  and increased concentration of 17-ketosteroids in the urine  were  revealed.  What  disease  is the most likely?

Explanation

       17-ketosteroids (17-KS) are substances that are formed when the body breaks down male steroid sex hormones called androgens and other hormones released by the adrenal glands in males and females; and by the testes in males. Increased levels of 17-ketosteroids may be due to: adrenal gland problems such as tumor – cushing’s syndrome, cushing’s disease; ovarian or testicular cancer. Itsenko-cushing disease is hypercortisolism due to excess pituitary secretion of adrenocorticotropic hormone (ACTH). Excess ACTH leads to excess adrenal hormones → excess mineralocorticoids; excess sex hormones (↑17-KS); excess glucocorticoids (steroid diabetes). Steroid diabetes is a medical term referring to prolonged hyperglycemia due to glucocorticoid (cortisol) excess. Steroid diabetes → 17-KS.

94. A   32-year-old   patient  consulted   a doctor   about   the   absence   of  lactation after  parturition. Such disorder might be explained by the  deficit of the  following hormone:

Explanation

Prolactin is responsible for lactogenesis. Prolactin is secreted from anterior pituitary gland. It is necessary for the final preparation of mammary glands for the production and secretion of milk. Lactation means synthesis, secretion and ejection of milk.
95.

An infant has pylorospasm, weakness, hypodynamia, convulsions  as a result  of frequent vomiting. What kind of acid-base disbalance  is it?

Explanation

Acidosis is the reduction in pH (increase in H+ concentration) below normal range. pH is less than 7.35; it is produced by:

·        Increase in partial pressure of CO2 in the body.

·        Decrease in HCO3- concentration.

Alkalosis is the increase in pH (decrease in H+ concentration) above normal range. pH is greater than 7.45; it is produced by:

·        Decrease in partial pressure of CO2 in the body.

·        Increase in HCO3- concentration.

Each of these two disorders has respiratory and non-respiratory forms. The non-respiratory form is divided into metabolic and excretory(non-gaseous).

·        Respiratory acidosis is the acidosis that is caused by alveolar hypoventilation e.g. airway obstruction due to bronchitis or lung diseases (pneumonia).

·        Respiratory alkalosis is caused by alveolar hyperventilation e.g. hypoxia in high altitude.

·        Non-respiratory:

-Metabolic acidosis is characterized by excess accumulation of organic acids such as lactic acid, ketoacids and uric acid formed by normal metabolism e.g. in Diabetes mellitus or extreme/prolonged exercise.

-Excretory/Non-gaseous acidosis may develop in impaired renal H+ excretion related to increased loss of bicarbonate in urine; diarrhea causes acidosis by the loss of bicarbonate with faeces.

-Excretory/Non-gaseous alkalosis: vomiting (loss of gastric acid), increased metabolism of lactate and citrate (turns into bicarbonate and water), long-term use of thiazides and loop diuretics.

      It is excretory/non-gaseous alkalosis because of the frequent vomiting.

96. A weightlifter  has a disruption of thoracic  lymphatic  duct  as a result  of lifting a weight. Choose  the most likely site of injury:

Explanation

krushkrok No96 (2011)

The thoracic aorta is a continuation of the aortic arch. It resides within the posterior mediastinum next to the vertebral column. The thoracic aorta passes through the aortic hiatus to become continuous with the abdominal aorta. Other neighboring organs are the thoracic duct (found on the left), the azygos and hemiazygous veins and the left sympathetic trunk. The thoracic duct resides anterior to the vertebral column, in between the aorta and the azygos vein and posterior to the esophagus. In the course of weight lifting there is increased arterial blood pressure which stretches the thoracic aorta and this can compress the thoracic duct while passing through the aortic hiatus.

Aortic hiatus is one of the openings of the diaphragm which resides in the middle between the right and left crura. Above, it is bounded by the median arcuate ligament that prevents the aorta from being constricted. Beside the aorta, the thoracic lymphatic duct also passes through the aortic hiatus. Other openings of diaphragm includes the esophageal hiatus for esophagus; caval opening for inferior vena cava.

97.

A woman suffering from osteochondrosis has acute pain in her humeral articulation that  gets worse  when  she tries  to abduct her  shoulder. These  symptoms  might  be caused by damage of the following nerve:

Explanation

krushkrok No28 (2013)

Axillary nerve innervates the muscles (e.g. deltoid) that abducts the upper limb.

Deltoid muscle:

Origin: lateral third of clavicle, acromion of scapula, spine of scapula.

Insertion: Deltoid tuberousity of humerus.

Innervation: Axillary nerve ( a terminal branch of branchial plexus receiving fibres from C5 & C6 ventral rami.

Action:

Anterior part- flexion & medial (internal) rotation of the humerus at glenohumeral joint.

Middle part- abduction of the humerus at the glenohumeral joint.

Posterior part – extension & lateral (external) rotation of the humerus at the glenohumeral joint.

98.

Pyeloureterography X-ray photo showed  a renal  pelvis with minor  calyces only  (major  calyces  were  absent). What form of urinary tracts of a kidney was revealed?

Explanation

     The embryonic period or period of organogenesis occurs from the 3rd-8th weeks of development. The kidney come from 3 slightly overlapping kidney systems – pronephros, mesonephros and metanephros. Metanephros is the definitive kidney, it appears in the 5th week. Collecting ducts of the permanent kidney develop from the ureteric bud, the bud dilates forming the primitive renal pelvis. These buds continue to subdivide until 12 or more generations of tubules have formed. The tubules of the second order enlarge and absorb those of the 3rd and 4th generations, forming the minor calyces of the renal pelvis. During further development, collecting tubules of the 5th and successive generations elongate considerably and converge on the minor calyx, forming the renal pyramid. The ureteric bud gives rise to the ureter, the renal pelvis, major and minor calyces.

99. A female patient presents with endocrine dysfunction   of  follicular  cells of  the   ovarian   follicles  resulting   from an inflammation. The synthesis of the following hormone will be inhibited:

Explanation

       Sexual cycles are controlled by the hypothalamus. Gonadotropin-releasing hormone (GnRH), produced by the hypothalamus acts on cells of the anterior lobe (adenohypophysis) of the pituitary gland which in turn secrete gonadotropins. These hormones, follicle-stimulating hormone (FSH) and luteinizing hormone (LH), stimulate and control cyclic changes in the ovary. At the beginning of each ovarian cycle, 15-20 primary stage (preantral) follicles are stimulated to grow under the influence of FSH. Under normal conditions, only one of these follicles reaches full maturity and only one oocyte is discharged; the others degenerate and become atretic. FSH also stimulates maturation of follicular (granulosa) cells surrounding the oocyte. In cooperation, theca interna and granulose cells produce estrogens. As a result of this estrogen production, the uterine endometrium enters the follicular or proliferative phase. Thinning of the cervical mucus occurs to allow passage of sperm and the anterior lobe of the pituitary gland is stimulated to secrete LH. So dysfunction of follicular cells will inhibit the production of estrogen.

100.

A  19-year-old   female   patient  has had low haemoglobin rate  of 90-95 g/l since childhood.  Blood  count  results obtained  after  hospitalisation are  as  follows: erythrocytes  -  3, 2 ·  1012/l,  Hb-   85  g/l, colour index - 0,78; leukocytes  - 5, 6 · 109/l, platelets - 210 · 109/l. Smear  examination  revealed   anisocytosis,   poikilocytosis and  target  cells. Reticulocyte rate  is 6%. Iron  therapy was ineffective.  What  blood pathology corresponds with the described clinical presentations?

Explanation

     Thalassemia or target cell anemia or Cooley’s anemia is an hemoglobinopathy (abnormal hemoglobin in RBCs). Defect in α-globin chain results in α thalassemia; and point mutations in β-globin chain results in β thalassemia. It is characterized microscopically by anisocytosis, poikilocytosis, target cells and is characterized by increased hemolysis, enlargement of spleen and liver, progressive anemia with erythroblastemia (reticulocyte). It’s a microcytic, hypochromic anemia.

Sickle cell anemia is also a hemoglobinopathy (abnormal hemoglobin in RBCs). It results as a result of a point mutation causing a single amino acid replacement in β chain (substitution of glutamic acid with valine) to produce hemoglobin S (HbS). The condition is characterized by the presence of sickle-like erythrocytes revealed during crisis, they cause stasis, hemorrhages and infarctions.

101.

A  liquidator of  a  breakdown  at  a nuclear  power  plant  who  was irradiated complained about  vomiting that occurs all of a sudden.  What  medication should  be prescribed?

Explanation

         Metoclopramide is an antiemetic drug; an antidopamine. It blocks D2-dopamine receptors of emetic center in the CNS. It has a central antiemetic effect in vomiting of any genesis. Uses – vomiting induced by irritation of gastrointestinal mucous membrane, post-operative period, pregnancy toxicosis, radiation sickness etc.

102. A patient with acute myocardial infarction has been administered heparin as   a   part   of   complex   therapy.  Some time   after   heparin  injection   the   patient  developed hematuria. What  heparin antagonist should  be injected  in order  to manage the complication?

Explanation

 

Protamine sulphate antagonizes the anticoagulant effects of heparin. The positively charged protamine interacts with the negatively charged heparin forming a stable complex without anticoagulant activity.

Heparin is a natural anticoagulant produced in mast cells and basophils. It is an injectable, rapidly acting anticoagulant that is often used acutely to interfere with the formation of thrombi. Heparin is used in the prevention of venous thrombosis and the treatment of a variety of thrombotic diseases such as pulmonary embolism and acute myocardial infarction. Heparin binds to antithrombin III, with the subsequent rapid inactivation of coagulation factors. Antithrombin III inhibits serine proteases, including several of the clotting factors, most importantly, thrombin (factor IIa) and Factor Xa (a-active).  
103.

A 45-year-old patient was admitted to the  cardiological  department. ECG  data: negative  P wave overlaps  QRS  complex, diastolic interval  is prolonged after extrasystole. What  type  of extrasystole is it?

Explanation

           Nodal or atrioventricular (AV) extrasystole is caused by stimulus arising from AV node. The excitation of the atria differs from normal, the impulse is transmitted retrogradely from bottom to top. The following signs are characteristic of the ECG in nodal extrasystole:

·        Premature appearance of the cardiac complex.

·        Changes in the P wave which becomes negative to show the retrograde atrial excitation (in some cases the P wave is absent on the ECG).

·        If the atria and ventricles are excited synchronously, the P wave superimposes (overlaps) the QRS complex.

104.

A  patient complains  of hydruria (7 liters per day) and polydipsia.  Examination  reveals  no  disorders of carbohydrate metabolism.  These   abnormalities  might be caused by the dysfunction  of the following endocrine gland:

Explanation

ADH (vasopressin) is produced in neurohypophysis (posterior pituitary gland). ADH causes reabsorption of free water through aquaporins in collecting ducts of kidney. Diabetes insipidus (D.I.) is characterized by intense thirst and polyuria (hyperdiuresis) with inability to concentrate urine due to lack of ADH (central D.I.) or failure of response to circulating ADH (nephrogenic D.I.). in central D.I. antidiuretic hormone (ADH) is decreased; in nephrogenic D.I. ADH is normal. Therefore, the hormonal drug we can use to treat diabetes insipidus is to replace the low antidiuretic hormone (ADH) in central diabetes insipidus. So, we use ADH – a posterior pituitary hormone which increases water reabsorption in renal tubules and decrease dieresis. Its analogue desmopressin acetate can also be administered intranasally. Nephrogenic diabetes insipidus does not require hormonal replacement.  
105.

Before  a surgery a blood sample of a 30-year-old  man  has  been  typed.  Blood is Rh-positive. Standard  serums  of  such groups as 0 αβ (I), Аβ (II), Вα (III) didn’t activate  erythrocyte agglutination reaction. The group of the analyzed blood is:

Explanation

Blood group O(I): no antigens, therefore no agglutination.

Blood group A(II): A antigen, agglutinate with blood group B(III) and O(I).

Blood group B(III): B antigen, agglutinate with blood group A(II) and O(I).

Blood group AB(IV): A and B antigen, agglutinate with all blood groups. No antibody.

Blood group name is determined by the antigen present on RBC, but the patient has an opposite antibody. So whenever, the antibody corresponds to the antigen, there is agglutination. Since O does not have any antigen, no agglutination can occur.

106.

Medical  examination of  a  20-year- old woman revealed  a dense encapsulated node  1 cm in diameter that  was palpated in the  mammary gland.  The  postoperative biopsy revealed  connective tissue overgrowth around  the  mammary ducts and glandular components of different  diameter that didn’t make lobules  and bore no signs of cellular  abnormality. What  diagnosis will be made?

Explanation

FullSizeRender (21)

Adenoma: benign epithelial tumor from the epithelium of the glands and glandular organs. More often they can be found in the breast, thyroid gland, liver, ovaries, prostate gland, GIT. In the question, it is located in the fibrous stroma – FIBROADENOMA.

107. A 38-year-old  male patient has been ill with systemic lupus  erythematosus for three years. He was diagnosed with diffuse renal   affection   accompanied  by  massive  edemata and  expressive   proteinuria. What  is the  most  likely  cause  of proteinuria development?

Explanation

Systemic lupus erythematous (SLE, Libman-sacks disease) is the classic prototype of the multisystem disease of autoimmune origin, characterized by a bewildering array of autoantibodies, particularly antinuclear antibodies. It is characterized principally by injury to the skin, joints, kidney, and serosal membranes. Antinuclear antibody is directed against several nuclear antigens and can be grouped into 4 categories:

·        Antibodies to DNA

·        Antibodies to histones

·        Antibodies to nonhistone proteins bound to RNA

·        Antibodies to nuclear antigens.

SLE is a type III hypersensitivity reaction (autoimmune - immune complex) with formation of immune complexes. It can cause diffuse proliferative glomerulonephritis seen under the light microscope as “wire loop” of capillaries and granular under the immunofluorescence microscopy.

108.

A  student failed  to  answer  all  the questions  of examination paper  correctly. As   a  result   he   blushed,   felt   hot   and lost confidence. What type of arterial hyperemia has developed in this case?

Explanation

·        Neuroparalytic arterial hyperemia is caused by damage or blockage of α-adrenoreceptors (sympathetic nervous system). It is characterized by reduction or absence (paralysis) of the sympathetic nervous system effects on the walls of the arteries and arterioles.

·        Neurotonic arterial hyperemia is caused by activation of parasympathetic nervous system; irritation of vascular dilators part of vascular center (CNS) or inhibition of vascular – constrictor part of this center (vasomotor center of CNS); M-cholinoreceptors; H2-histaminereceptors. It is characterized by predominance of the parasympathetic nervous system effects on arterial vascular walls.

After the exam parasympathetic nervous system is the dominant system. Before the exam, sympathetic nervous system is the dominant system.

109.

Acute    renal    impairment   caused death  of a bleeding patient. Autopsy revealed  enlarged kidneys  with  a  broad pale pink cortical layer expressively demarcated from  dark  red  renal  pyramids.   Macroscopic    examination  revealed lack of epithelial nuclei of convoluted tubules,  tubulorrhexis,  phlebostasis. The cell nuclei of choroid  glomus and straight tubules  were  present. What  pathology is it?

Explanation

          Acute tubular necrosis/Necrotic nephrosis/Necronephrosis involves the death of tubular epithelial cells that form the renal tubules of the kidneys. Most common cause of acute renal failure. It can be ischemic or nephrotoxic.

·        Ischemic acute tubular necrosis occurs due to hypoperfusion of the kidneys.

·        Nephrotoxic acute tubular necrosis occurs as a result of direct damage to tubular cells by ingestion, injection or inhalation of a number of toxic agents. Toxic agents causing nephrotoxic acute tubular necrosis includes mercuric chloride, ethylene glycol, carbon tetrachloride etc.

Macroscopically, the kidneys are enlarged and swollen (edematous). On cut section, the cortex is pale, while the medulla is slightly darker than normal. The capsule can be easily removed.

110.

A female  patient consulted  a doctor about   pain   and   limited   movements  in the   knee   joints.  Which  of  the  following nonsteroid anti-inflammatory drugs should  be administered taking into consideration that  the  patient has a history  of chronic gastroduodenitis?

Explanation

The nonsteroidal anti-inflammatory drugs (NSAIDs) are a group of chemically dissimilar agents that differ in their antipyretic, analgesic and anti-inflammatory activities. They act primarily by inhibiting the cyclooxygenase (COX 1 and 2) enzymes that catalyze the first step in prostanoid biosynthesis. This leads to decrease prostaglandin synthesis with both beneficial and unwanted effects. Aspirin is one of the most important NSAID. Its most common side effect is its gastrointestinal (GI) effect. Normally, prostacyclin (PGI2) inhibits gastric acid secretion, whereas PGE2 and PGF stimulate synthesis of protective mucus in both the stomach and small intestine. In the presence of aspirin, these prostanoids are not formed, resulting in increased gastric secretion and diminished mucus protection. This may cause epigastric distress, ulceration, hemorrhage and iron-deficiency anemia.

Celecoxib is a selective cyclooxygenase 2 (COX-2) inhibitor. Approved for treatment of rheumatoid arthritis, osteoarthritis, acute to moderate pain, also approved for patients with peptic ulcers or gastroduodenitis.

111.

A patient presents with icteritiousness of skin, scleras and mucous membranes. Blood  plasma  the  total  bilirubin is increased,  stercobilin is increased in  feces,  urobilin   is  increased   in  urine. What type of jaundice  is it?

Explanation

Hemolytic/Prehepatic Jaundice is the type of Jaundice that occurs because of excessive destruction of RBCs resulting in ↑ blood level of free, Indirect, unconjugated bilirubin. In this condition, the excretory function of the liver is normal. But the quantity of bilirubin ↑ enormously. The liver cells cannot excrete that much excess bilirubin rapidly. Formation of Urobilinogen (stercobilin) ↑ resulting in the excretion of more amount of stercobilin in stool and urine.

Indirect; Hemolytic; Prehepatic

Mixed; Parenchymal; Hepatic

Direct; Obstructive; Mechanic; Posthepatic

Stercobilin (faeces)

        ↑↑↑

Decreases (pale faces)

Absent (clay coloured faeces)

Type of bilirubin in blood

Unconjugated

Conjugated and Unconjugated

Conjugated

·       Gilbert syndrome (familial nonhemolytic Jaundice): Autosomal recessive defect. Impaired UGT activity. Jaundice occurs with fasting, volume depletion, stress, menses.

112.

Blood   plasma   of  a  healthy   man contains  several dozens of proteins.  During an illness new proteins  can originate, namely   the   protein  of  \\\"acute   phase\\\". Select such protein from the listed below:

Explanation

     Acute phase reactants are factors whose serum concentrations change significantly in response to inflammation produced by the liver in both acute and chronic inflammatory states. Notably induced by IL-6 e.g. C-reactive protein, hepcidin, serum amyloid A are all upregulated; while albumin and transferring are downregulated. C-reactive protein is an opsonin, it fixes complement and facilitates phagocytosis. It is measured  clinically as a sign of ongoing inflammation.

113.

A patient working at a pig farm complains    about    paroxysmal    abdominal pain,  liquid  feces with admixtures of mucus and blood, headache, weakness, fever. Examination of large intestine revealed  ulcers  from 1 mm up to several cm large, feces contained oval unicellular organisms  with cilia. What disease should be suspected?

Explanation

IMG_9915        

 Balantidium coli causes balantidiasis. It is the only ciliated protozoan that causes human disease i.e. diarrhea. Domestic animals, especially pigs are the main reservoir for the organism and humans are infected after ingesting the cysts in food or water contaminated with animal or human faeces. Diagnosis is made by finding large ciliated trophozoites or large cysts with a characteristic V-shaped nucleus in the stool. The trophozoites excyst in the small intestine, travel to the colon (large intestine) and by burrowing into the wall cause an ulcer similar to that of Entamoeba histolytica.

114.

During  hypersensitivity test a patient got subcutaneous injection  of an antigen which  caused  reddening of skin,  edema, pain  as a result  of histamine action.  This biogenic amine  is generated as a result of transformation of the  following  histidine amino acid:

 

Explanation

image         

Certain amino acids undergo decarboxylation that means the removal of their α-carboxyl group resulting in liberation of CO2 and formation of biogenic amines. Biogenic amines are physiologically active substances such as hormones, neurotransmitters etc. decarboxylation of amino acids:

·        Tryptophan → Niacin → NAD+/NADP+

Tryptophan → Serotonin →Melatonin

·        Histidine → Histamine

·        Glutamine → GABA

Glutamine → Glutathione

115. During fighting a man had a cardiac arrest as a result of a hard blow to the upper  region  of anterior abdominal wall. Which of the described  mechanisms might have provoked the cardiac arrest?

Explanation

Reflex activity is the response to a peripheral nervous stimulation that occurs without our consciousness. It can be:

* Inborn or unconditioned: present from birth, does not require previous learning or training.

* Acquired or conditioned: developed after conditioning or training. Acquired after birth.

Goltz reflex (press or blow to the epigastric region) → ↓heart rate (parasympathetic effect).

Aschner’s reflex (press on eyeball) → ↓heart rate. This is mediated by nerve connections between the ophthalmic branch of trigeminal cranial nerve via the ciliary ganglion and the vagus nerve of parasympathetic nervous system.

116.

There  is a severe time restriction for people’s  staying  at  a height  of over  800 m  above   the  sea  level  without   oxygen bombs.  What  is the  life limiting factor  in this case?

Explanation

            Although the percentage of oxygen in inspired air is constant at different altitudes; the fall in atmospheric pressure at higher altitudes decreases the partial pressure of inspired oxygen and the driving pressure for gas exchange in the lungs. A fall in inspired oxygen pressure reduces the driving pressure for gas exchange in the lungs and in turn produces a cascade of effects right down to the level of the mitochondria, the final destination of the oxygen. Partial pressure is used to describe a mixture of gases. It’s a way of describing how much of a gas is present. It is the pressure that any one gas would exert on the walls of the container if it were the only gas present.

117.

A patient with marked pneumofibrosis   that    developed   after    infiltrating  pulmonary tuberculosis has  been  diagnosed  with respiratory failure.  What  is its pathogenetic type?

Explanation

Restrictive respiratory disease is the abnormal respiratory condition characterized by difficulty in inspiration. E.g poliomyelitis,myasthenia gravis, paralysis of diaphragm, spiral cord diseases, pleural effusion, fibrosis. (lung fibrosis-pneumofibrosis). Obstructive respiratory disease is the abnormal respiratory condition characterized by difficulty in expiration. E.g asthma,chronic bronchitis, emphysema, cystic fibrosis.       
118.

During surgical manipulations a patient has been given novocaine  injection for anesthesia. 10 minutes  later  the  patient developed paleness,  dyspnea,  hypotension. What type of allergic reaction is it?

Explanation

Considering the time (10mins) – immediate

Type I Hypersensitivity reaction (HSR); anaphylactic and atopic: free antigen cross-links IgE on presensitized (i.e. exposed to the antigen before) mast cells and basophils, triggering immediate release of vasoactive amines that act at postcapillary venules (i.e. histamine). Reaction develops rapidly after antigen exposure because of preformed antibody from first exposure. IgE is the main immunoglobulin involved in type I HSR. Type I: uses IgE and IgG4

119.

A sensitive  neural  ganglion  consists of roundish neurocytes with one  extension that divides into axon and dendrite at a some distance  from the perikaryon. What are these cells called?

Explanation

image           

Neurons are classified on the basis of the number of processes extending from the cell body:

·        Pseudounipolar: neurons have one process (extension), which divides close to the cell body into two long processes (axon and dendrite). The vast majority are located in the dorsal root ganglia and cranial nerve ganglia.

·        Unipolar neurons have only one pole. From a single pole, both axon and dendrite arise. This type of nerve cells is present only in embryonic stage in human beings.

·        Bipolar neurons have two poles – one axon and one dendrite.

·        Multipolar neurons have many poles – one axon and two or more dendrites.

120. A patient complained about  dizziness, memory  impairment, periodical convulsions. It was revealed that these changes were caused by a product of decarboxylation of  glutamic  acid.  Name this product:

Explanation

image          

Certain amino acids undergo decarboxylation that means the removal of their α-carboxyl group resulting in liberation of CO2 and formation of biogenic amines. Biogenic amines are physiologically active substances such as hormones, neurotransmitters etc. decarboxylation of amino acids:

·        Tryptophan → Niacin → NAD+/NADP+

Tryptophan → Serotonin →Melatonin

·        Histidine → Histamine

·        Glutamine → GABA

Glutamine → Glutathione

121.

It  was  found  out  that  some compounds, for instance  fungi toxins and some  antibiotics   can  inhibit   activity  of RNA-polymerase.  What  process  will be disturbed in a cell in case of inhibition  of this enzyme?

Explanation

     The process of transcription of a typical gene can be divided into 3 phases: initiation, elongation and termination. The process of transcription begins with the binding of the RNA polymerase holoenzyme to a regionof the DNA known as the promoter. RNA polymerase is a multisubunit enzyme that recognizes a nucleotide sequence (the promoter region) at the beginning of a length of DNA that is to be transcribed. It makes a complementary RNA copy of the DNA template strand and then recognizes the end of the DNA sequence to be transcribed (the termination region).

122.

During  an  experiment the  myotatic reflex has been studied in frogs. After extension in  a  skeletal  muscle  its reflectory contraction was absent. The reason  for  it  might  be  a  dysfunction   of the following receptors:

Explanation

          Muscle spindle s a spindle shaped proprioceptor situated in the skeletal muscle. It is formed by modified skeletal muscle fibers called intrafusal muscle fibers. Muscle spindle gives response to change in the length of the muscle. It detects how much the muscle is being stretched and sends this information to the CNS via sensory nerve fibers. Stimulation of muscle spindle elicits the stretch reflex. Extensor muscles, particularly the antigravity muscles exhibit a severe and prolonged contraction during stretch reflex. Stretch reflex is the reflex contraction of muscle when it is stretched. Also called myotatic or a monosynaptic reflex. It is the quickest of all the reflexes.

123.

An adult man presents with systemic arterial  pressure  drop   from   120/70  to  90/50  mm   Hg.   This   resulted  in  reflex vasoconstriction. Vasoconstriction will be  minimal in the following organ:

Explanation

     The drop in arterial pressure (AP) will cause a reflex tachycardia, the heart increases its contractility and cardiac output. Vasoconstriction will be minimal in the heart, because the heart needs to pump more i.e. do more work. Therefore, it needs more blood supply. But vasoconstriction will be maximal in the GIT (bowels), muscles, skin due to centralization of blood flow (i.e. redistribution of blood to important organs like heart, brain)

124.

A culture  of monkey  cells (Vero) and a group of mouse  sucklings were infected with an inoculum  taken  from a child with provisional  diagnosis  \"enterovirus infection\".  There  was  no  cytopathic effect  on the cell culture  but mouse  sucklings died. What enteric viruses might have caused disease of this child?

Explanation

       Picornaviruses are small nonenveloped viruses composed of an icosahedral nucleocapsid and a single-stranded RNA genome (ssRNA). Picornavirus family ncludes two groups of medical importance: the enteroviruses and rhinoviruses. Among the major enteroviruses are the poliovirus, coxsackieviruses, echoviruses and hepatitis A virus.

Coxsackieviruses are named for the town of Coxsackie, NY, where they were first isolated. Coxsackievirus cause a variety of diseases. Coxsackieviruses are divided into group A and group B viruses based on early observations of their pathogenicity in newborn mice (mouse sucklings). Group A causes flaccid paralysis while group B causes spastic paralysis. Coxsackie A virus causes paralysis and death of the mice; coxsackie B causes less severe infection in the mice.

125. An electron microphotography of a fragment of proper gastric gland shows a big irregular round-shaped cell. There are  a lot of intracellular tubules  and  mitochondria in the  cytoplasm.  Specify this cell:

Explanation

Parietal (oxyntic) cells are found in the neck of the fundic glands, among the mucus neck cells and in the deeper part of the gland. They secrete HCl and intrinsic factor. When examined with the transmission electron microscope they are seen to have an extensive intracellular canalicular system that communicates with the lumen of the gland. Also an elaborate tubulo-vesicular membrane system is present in the cytoplasm adjacent to the canaliculi. Numerous mitochondria with complex cristae and many matrix granules supply the high levels of energy necessary for acid secretion.

126.

Blood   count   of  an   athlete  is  as follows:  erythrocytes  -  5, 5 · 1012/l,  Hb - 180 g/l, leukocytes  - 7 . 109/l, neutrophils - 64%, basophils - 0,5%, eosinophils - 0,5%, monocytes  - 8%, lymphocytes  - 27%. First of all, such results indicate the stimulation of:

Explanation

 

        Normal range

               Result

RBC (erythrocytes): Men

      4.2 – 5.2 X 1012/L

          5.5 X 1012/L     (↑)

                               Women

      3.7 – 4.7 X 1012/L

 

Hemoglobin:  Men

      135 – 165g/L

           180g/L        (↑)

                        Women

      120 – 140g/L

           

Leukocytes (WBC):

      4.0 – 9.0 X 109/L

      7 X 109/L    (normal)

    Basophils

         0 - 1%

           0.5%    (normal)

    Eosinophils

         0.5 – 5%

           0.5%    (normal)

    Neutrophils

         47 – 72%

           64%     (normal)

    Lymphocytes

         19 – 37%

           27%     (normal)

    Monocytes

           3 – 11%

            8%      (normal)

 

This results indicates erythropoiesis i.e. synthesis of RBCs.

127.

A patient died from cardiopulmonary decompensation. Histological  examination revealed  diffused pulmonary affection along  with interstitial edema,  infiltration of tissue by limphocytes,  macrophages, plasmocytes;  pulmonary fibrosis,  panacinar  emphysema. What  is the  most  likely diagnosis?

Explanation

          Fibrosing alveolitis is also known as interstitial pneumonia. It is characterized by progressive scarring of both lungs. The scarring (fibrosis) involves the supporting framework (interstitium) of the lung. There are patchy or lobar areas of congestion without the consolidation of bacterial pneumonias. A predominance of interstitial pneumonitis with widened, edematous alveolar walls containing a mononuclear inflammatory cell infiltrates. Interstitial pneumonia is the most common form of interstitial fibrosis.

128.

If a man  has  an  attack  of bronchiospasm it is necessary to reduce  the effect of vagus  on  smooth  muscles  of bronchi. What  membrane cytoreceptors should  be blocked for this purpose?

Explanation

Muscarinic receptors belong to the class of G protein-coupled receptors. These receptors, in addition to binding Acetylcholine, it also recognizes muscarine – an alkaloid that is present in certain poisonous mushrooms. Muscarinic effects include miosis, spasm of accommodation, increasing of lacrimal, salivary, gastrointestinal, bronchial and sweat glands secretion, increase of smooth muscle tonus of internal organs, bradycardia, decreased arterial pressure.

M-cholinoreceptors increase muscle tonus of internal organs. Therefore blocking this receptor will relieve the bronchospasm.

   
129.

A    male    patient   has    been    diagnosed  with  gastric  ulcer.  Bacteriological examination of biopsy  material from the   affected   part   of  stomach   revealed small colonies of gram-negative, oxide reductase-positive flexibacteria  that  grew on  the  chocolate  agar  on  the  fifth  day. Which  of  the  following  microorganisms is the most likely causative agent?

Explanation

          Helicobacter pylori causes gastritis and peptic ulcers. Infection with H. pylori is a risk factor for gastric carcinoma and is linked to mucosal-associated lymphoid tissue (MALT) lymphoma. It is a gram negative, flexibacteria, oxidase positive, microaerophilic, motile and the only species in the Helicobacter genus to have multiple unipolar – sheathed flagella. Microaerophiles need O2 because they cannot ferment or respire anaerobically. However, they are poisoned by high concentrations of O2. They gather in the upper part of the test tube but not the very top. CO2 is essential for initial growth of H. pylori in liquid media (microaerophilic property). They produce urease, so it’s a diagnostic tool and not a necessary consideration for cultivation (culture medium).

130.

There     are     several     groups     of molecular mechanisms  playing important part   in  pathogenesis  of  insult   to  cells which contributes to the pathology development. What processes are stimulated  by  proteinic   damage   mechanisms?

Explanation

            Protein mechanism is one of the molecular mechanisms of cell injury. The protein mechanism of cell damage includes:

·        Inhibition of enzymes (reversible and irreversible)

·        Denaturation – violation of native structure of proteins

·        Proteolysis – that is carried out under the action of lysosomal enzymes.

131.

A  patient has  been  diagnosed   with  a  compression fracture  of  a  lumbar vertebra.  As  a  result   he  has  a  considerable  increase    in   curvature   of   the lumbar  lordosis.  Which ligament  damage can induce such changes in the spine curvature?

Explanation

Lordosis is an exaggerated anterior curvature of the spine, most often lumbar. Lordosis is present in the cervical and lumbar regions (cervical and lumbar lordoses). The intervertebral synchondroses and symphyses are reinforced by the longitudinal ligaments which run along the entire spine.

·        Anterior longitudinal ligament is a band which extends from the atlas to the pelvic surface of the hip bone along the anterior surfaces of the vertebral bodies.

·        Posterior longitudinal ligament runs along the posterior surface of the vertebral bodies (in the vertebral canal).

Therefore, a damage to the anterior longitudinal ligament can induce lordosis.

132.

A 49-year-old  patient with croupous pneumonia died from pneumococcal septicemia. Autopsy  revealed  up  to  700 ml of turbid greenish-yellow foul-smelling liquid in the left pleural cavity. The pleural leaflets   were   dull  and   plethoric.  What form of pleural inflammation is it?

Explanation

        Complications of croupous or lobar pneumonia includes empyema, gangrene, carnification, abscess formation, bacteremic spread which leads to purulent meningitis, bacterial endocarditis, arthritis, pericarditis and other organs.

     Empyema is a purulent inflammation of serous membranes (empyema of pleura, empyema of gall bladder and urinary bladder etc). Purulent or suppurative inflammation is characterized by the production of large amounts of pus or purulent exudates (liquids) consisting of a lot of neutrophils, necrotic cells and edema fluid.

 
133.

A 35-year-old  man  developed acute heart failure while running for a long time. What changes in ionic composition can be observed  in the cardiac muscle?

Explanation

Increase in cardiac workload can cause imbalances in oxygen (O2) supply and demand to the heart itself. Blood supply to the heart occurs during diastole. Increase heart rate (↑HR) → ↓diastolic period → ischemia in myocardium. Ischemia, hypoxia, heart failure are associated with disruptions in intracellular Na+ and Ca+ concentration homeostasis of myocardial cells. A decrease efflux or increase influx of Na+ may cause cellular Na+ overload. Na+ overload is followed by an ↑influx of calcium (Ca2+) through Na+-Ca2+ exchanger. Failure to maintain the homeostasis of Na+ and Ca+ leads to electrical instability (arrhythmias), mechanical dysfunction (reduced contractility and ↑ diastolic tension) and mitochondrial dysfunction. These events increase ATP hydrolysis and decrease ATP formation and if left uncorrected, they cause cell injury and death → acute heart failure.

134.

A     child     with     a    history     of frequent   angine    and    pharyngitis    has been   diagnosed   with  lymphadenopathy and splenomegaly. His appearance is characterised by  pastosity  and  paleness, muscular  tissue is poorly developed. Lymphocytosis is present. What  kind  of diathesis is it?

Explanation

Abnormality of constitution, so called diathesis is characterized by pathological reactions on physiological agent. Classification of diathesis:

·        Hemorrhagic: characterized by hemorrhagic reaction to physiological factors.

·        Thymicolymphatic/Lymphohypoplastic: characterized by enlargement of lymphatic nodes, muscular atrophy, individual pale, pastous, predisposing to autoallergic disease, lymphocytosis, angina and other infectious diseases.

·        Neuroarthritic/Gouty: predisposing to arthralgia, arthritis, rheumatism, obesity, gout, psychic disease.

·        Edematic/Exudative: characterized by edema reaction to different environmental factors.

·        Asthenic: expressed in hypodynamia, hypotonia.

135.

In response to a change in body position from horizontal to vertical blood circulation system develops reflectory pressor  reaction. Which  of the  following is its compulsory  component?

Explanation

From horizontal (lying position) to vertical (standing position) will make all blood flow towards the lower extremities, thereby reducing venous return (blood going upwards towards the heart) in the first place. This will definitely ↓ cardiac output (C.O) → ↓blood supply to brain (this can lead to fainting). Then there is reflex vasoconstriction to increase venous return to heart, in the absence of any pathology.

136.

After    an    immunoassay   a    child   was   diagnosed    with   immunodeficiency  of  humoral immunity.  What  is the reason  for the primary  immunodeficiency development in the child?

Explanation

     The humoral immunity deficiency is the most frequent among primary immunodeficiency types. The genetic defects of immune system development are on the basis of primary immunodeficiency, causing various clinical symptoms. There are 36 primary immunodeficiency types according to the last international classification, which are divided into 5 groups depending on the kind of deficiency of the immune system:

·        Deficiency of humoral immunity (B-component): frequently in bacterial infections.

·        Deficiency of cellular immunity (T-component): frequently in viral infections.

·        Combined T and B – immunodeficiency: during both bacterial and viral infections.

·        Deficiency of phagocytes system.

·        Deficiency of complement system: in gram negative coccal infections.

137. Life   cycle  of  a  cell  includes   the process  of DNA  autoreduplication. As a result of it monochromatid chromosomes turn  into  bichromatid ones.  What  period of  cell  cycle  does  this  phenomenon fall into?

Explanation

Somatic cell division is a cyclic process divided into two phases: mitosis (M phase) and interphase. Three other phases, Gap 1 (G1); synthesis phase (S) and Gap 2 (G2) further subdivide interphase. M phase is followed by G1.

M phase: karyokinesis – division of the nucleus into two daughter nuclei.

                  Cytokinesis – division of the cell into two daughter cells.

G1: a period in which no DNA synthesis occurs. Period of cell growth S or DNA synthesis phase follows G1 phase. The DNA of the cell is doubled. The S phase is followed by a period in which no DNA synthesis occurs, a second gap or G2 phase. A cell that leaves the cycle in G1 phase to begin “terminal” differentiation enters the G0 phase, (“O” stands for “outside” the cycle).

 
138.

Examination    of    a    12-year-old boy with developmental lag revealed achondroplasia: disproportional constitution  with  evident  shortening of  upper and lower limbs as a result of growth disorder of epiphyseal cartilages  of long tubal bones. This disease is:

Explanation

Autosomal dominant: often due to defects in structural genes. Many generations, both male and female are affected. Found in every generation; no generation is left out (skipped). Parent – child in every generation.

Autosomal recessive is usually seen in some generations (other generations are skipped).

It is sex linked, if only males or only females(X-linked) are affected.            

 X-linked dominant: it can be transmitted through both parents. Mothers transmit to 50% of daughters and sons; fathers transmit to all daughters but no son (father-daughters; no generation is left out).

X-linked recessive: sons of heterozygous mothers have a 50% chance of being affected. No male-to-male transmission. Recessive genetic diseases skips generations.

Y-linked: father to all sons; no female involvement (mother or daughter)

139.

2   days   after    labour    a   woman developed  shock  along  with  DIC syndrome  that caused her death.  Autopsy revealed  purulent endomyometritis, regional  purulent lymphangitis,  lymphadenitis and  purulent thrombophlebitis. There were also dystrophic alterations and interstitial inflammation of  parenchymal organs. What is the most likely diagnosis?

Explanation

           This observation describes sepsis. Septicemia is a form of sepsis. Hyperplasia of lymphoid and hemopoietic system is typical: the spleen is enlarged with pulp scraping (“septic spleen”). The lymph nodes are also enlarged. Interstitial inflammation develops in the parenchymal organs (heart, liver, kidneys). The stroma of organs is edematous; infiltration by neutrophils, lymphocytes and histiocytes is noted. It is also characterized by increased vascular permeability, fibrinoid changes in the vessels, allergic vasculitis that is responsible for hemorrhagic syndrome.

140.

After  a craniocerebral trauma a patient lost the ability to execute learned purposeful movements (apraxia). The injury is most likely localized in the following region of the cerebral cortex:

Explanation

         Apraxia is defined as a cognitive motor disorder in which the patient loses the ability to accurately perform learned, skilled actions. Apraxia is primarily a condition that localized to the dominant (usually left) hemisphere of the brain. In particular, lesions of supramarginal gyrus and underlying white matter of the left parietal lobe have been implicated to cause apraxia.

141.

Tissue inosytol triphosphates are generated as  a  result  of  the  phosphatidylinositol   diphosphate  hydrolysis   and act  as  secondary   agents   (mediators)  in the  mechanism  of hormone action.  Their effect in cells is directed at:

Explanation

         The protein hormones and catecholamines which act on a target cell are called first messenger. It combines with the receptor and forms hormone-receptor complex. Hormone-receptor complex activates the enzymes of the cell and cause the formation of the second messenger. The second messenger produces the effects of the first messenger inside the cell. Most common second messenger is cAMP; others are Ca2+ and calmodulin; inositol triphosphate; cGMP and diacylglycerol (DAG).

       Inositol triphosphate (IP3) is formed from phosphotidylinositol biphosphate (PIP3). Hormone-receptor complex activates the enzyme phospholipase which convert PIP2 into IP3. IP3 acts on protein kinase C and causes the physiological response by the release of calcium ions into the cytoplasm of target cell.

142.

A   patient  with   coronary    disease and  arrhythmia has been  administered a drug that blocks potassium channels and prolongs  the  action  potential. What  drug is it?

Explanation

           Class III antiarrhythmic drugs block potassium (K+) channels and thus diminish the outward potassium current during repolarization of cardiac cells. These agents prolong the duration of the action potential without altering phase O of depolarization or the resting membrane potential. Class III drugs include amiodarone, dronedarone, sotalol, dofetilide.

Corglyconum – short acting cardiotonic; it’s a cardiac glycoside (for acute heart failure).

Nitroglycerine – organic nitrate

Dobutamine – β1 agonist; it’s a nonglycoside cardiotonic

Lisinopril – angiotensin converting enzyme (ACE) inhibitor

143. A  patient takes  digoxin  for  cardiac insufficiency.  What  diuretic  may increase digoxin toxicity due to the intensified excretion of K + ions?

Explanation

IMG_9896        

 Hydrochlorothiazide is a thiazide diuretic. Thiazide diuretics induce hypokalemia and hyperuricemia in 70% of patients and hyperglycemia in 10% of patients. Serum potassium levels should be monitored closely in patients who are predisposed to cardiac arrhythmias (particularly individuals with left ventricular hypertrophy, ischemic heart disease or chronic heart failure) and those who are concurrently being treated with both thiazide diuretics and digoxin. In general, decreased serum levels of potassium predispose a patient to digoxin toxicity.

144.

During  starvation muscle proteins break up into free amino acids. These compounds will be the most probably involved into the following process:

Explanation

          Gluconeogenesis is making of glucose from non-carbohydrate sources i.e. fat and proteins. During a prolonged fast, hepatic glycogen stores are depleted and glucose is formed from precursors such as lactate, pyruvate, glycerol (derived from backbone of triacylglycerols) and α-ketoacids (derived from the catabolism of glucogenic amino acids). Amino acids derived from hydrolysis of tissue proteins (e.g. muscle proteins) are the major sources of glucose during a fast. α-ketoacids such as oxaloacetate and α-ketoglutarate are derived from the metabolism of glucogenic amino acids. Amino acids whose catabolism yields pyruvate or one of the intermediates of citric acid cycle (CAC) are termed glucogenic or glycogenic e.g. alanine, arginine, aspartate, histidine etc.

145.

A female  patient consulted  a doctor about a sense of epigastric discomfort, nausea  and anorexia. A duodenal content analysis   revealed    lamblia.   What   drug should be prescribed?

Explanation

Metronidazole forms toxic free radical metabolites in the bacterial cell that damage DNA. It is bactericidal, and an antiprotozoal. It is used to treat Giardia lamblia, Entamoeba, Trichomonas, Gardnerella vaginalis, Anaerobes (bacteroides, Clostridium difficile). It can be used with a proton pump inhibitor and clarithromycin for “triple therapy” against Helicobacter pylori.

146.

A  28-year-old   female  patient consulted a  gynecologist   about   sterility. Examination revealed  underdeveloped ovaries and uterus,  irregular menstrual cycle. Analysis of the sex chromatin revealed  2 Barr’s bodies  in most  somatic cells. What  chromosome disease  is most likely?

,

Explanation

Barr body is an inactive X-chromosome. A normal female has one barr body XX, a normal male has no barr body XY.

Trisomy XXXX (only one X is active in a female; therefore, 2 barr bodies)

KlinefelterXXY (one barr body)

Turner’s – XO (no barr body)

Edward and Patau involves autosomal chromosomes and not sex chromosomes.

147. A 1-year old baby has been hospitalized for body and limbs lesions. Examination  revealed  carnitine deficiency in the child’s muscles. A biochemical  reason  for this pathology is the disorder  of:

Explanation

image

The major pathway for catabolism of saturated fatty acids is a mitochondrial pathway called β-oxidation. After a long-chain fatty acid (LCFA) enters a cell, it is converted in the cytosol to its Co-A derivative. Because β-oxidation occurs in the mitochondrial matrix, the fatty acid must be transported across inner mitochondrial membrane which is impermeable to Co-A. therefore, a specialized carrier transports the long chain acyl group from the cytosol into the mitochondrial matrix. This carrier is carnitine and this rate-limiting transport process is called the carnitine shuttle. Since carnitine helps the mitochondria utilize energy, it plays a critical role in reducing the occurrence and impact of obesity. In addition to helping the mitochondria burn fat as energy, carnitine is also vital for removing waste products from mitochondria. Obesity and aging contribute to low carnitine levels, which compromises mitochondrial performance and increases insulin resistance, promoting further obesity and carnitine reduction.

148. During     anesthetization   a    patient presented with symptoms of tonus increase  of  parasympathetic nervous system such as hypersalivation and laryngospasm. What drug could have prevented these undesirable effects?

Explanation

Acetylcholine is a neurotransmitter. It increases salivary secretion and stimulates intestinal secretions and motility. It increases bronchial secretions, increases tone of detrusor urinae muscle, causing expulsion of urine, stimulates ciliary muscle contraction (accommodation). It increases exocrine gland secretions (e.g. lacrimal, salivary, gastric acid); gut peristalsis, bladder contraction, bronchoconstriction, papillary sphincter muscle contraction (miosis). Poisoning by cholinomimetics causes diarrhea, urination, miosis, bronchospasm, bradycardia, excitation of skeletal muscle and central nervous system, lacrimation, sweating and salivation. Cholinomimetics are cholinergic agonists which produce analogous effects of acetylcholine.

Atropine sulfate (M-cholinoblocker) blocks the acetylcholine response.  At low doses, it decreases cardiac rate (bradycardia). Arterial blood pressure is unaffected by atropine but at toxic levels, it will dilate the cutaneous vasculature. It is not effective in promoting healing of peptic ulcer. It can also relieve bronchial asthma.
149. Some  students  developed myodynia after  continuous physical  activity  during physical  education. The  reason  for  such condition was accumulation of lactic acid in the  skeletal  muscles.  It was generated in the students’ bodies  after  activation of the following process:

Explanation

       Athletes that are exercising intensely for the short periods of time, such as in a sprint race, build up large amounts of lactate in their muscles as the result of anaerobic glycolysis. The “warming down” period of continual movement under aerobic conditions performed by athletes for approximately 15mins after a race increases circulation and removes lactate from the muscles.

Lactate, formed by the action of lactate dehydrogenase (converting pyruvate to lactate) is the final product of anaerobic glycolysis in eukaryotic cells. In organs or cells that are poorly vascularized and/or lack mitochondria, formation of lactate is the major fate of pyruvate as seen in lens, cornea of the eye, kidney medulla, testes, leukocytes and red blood cells.

Aerobic glycolysis progresses to citric acid cycle from pyruvate. The cycle occurs totally in the mitochondria.

150.

A    newborn   develops    dyspepsia after   the   milk  feeding.   When   the   milk is substituted by the  glucose  solution the  dyspepsia  symptoms  disappear. The newborn  has the subnormal activity of the following enzyme:

Explanation

       Lactase (β-galactosidase) cleaves lactose (in milk) producing galactose and glucose. Hereditary deficiencies of lactase have been reported in infants and children with dissacharide intolerance. Treatment for this disorder is to reduce consumption of milk. This is seen when the milk is substituted by glucose solution.

151. A  doctor  asked  a patient to  breath out  fully  after  taking  a  normal   breath. What muscles contract during such exhalation?

Explanation

All muscles that elevate the rib cage are muscles of inspiration and those that depress the rib cage are muscles of expiration.

Muscles of inspiration:

·        Sternocleidomastoid: lift upward on the sternum.

·        Anterior serrati: lift many of the ribs.

·        Scalene: lift the first two ribs.

Muscles of expiration: Abdominal recti – pull down the lower ribs and other abdominal muscles also compress the abdominal contents upwards against the diaphragm and internal intercostals.

Normal respiration is accomplished by the movement of the diaphragm only.

152.

On an electron micrograph a scientist has identified a structure formed by  eight  histone  proteins and  a  part  of DNA molecule which makes about 1,75 revolutions around the molecules.  Which structure has been identified?

Explanation

image       

 Genes are contained in a complex of DNA and proteins (mostly histones) called chromatin and its basic unit of structure is the nucleosome. Each nucleosome is composed of an octamer (8) of histone proteins and approximately 140 base pairs of DNA. Nucleosomes themselves are joined into clusters by binding of DNA existing between nucleosomes (linker DNA) with other histone proteins. Nucleosomes keep the DNA tightly coiled such that it cannot be transcribed.

153.

A patient with massive burns developed acute renal insufficiency characterized    by    a    significant     and rapid  deceleration of  glomerular filtration. What is the mechanism  of its development?

Explanation

        Massive burns leads to hypovolemia and decreases general blood volume in circulation. This decrease in blood volume, decrease perfusion to all organs in the body. Decrease in renal perfusion (renal blood flow) will decrease glomerular filtration rate (GFR).

154.

A   three-year-old   child   has   had marked diarrhea for three  days. Immune electron microscopy  of his excrements revealed  bilayer pseudocovered capsid viruses that looked  like small spoke wheels. What viruses have been revealed?

Explanation

image

Rotavirus is the most important global cause of infantile gastroenteritis, is a segmented dsRNA virus (a reovirus). Major cause of acute diarrhea during winter, especially in day care centers, kindergartens. ‘Roundish structure like a wheel’

Adenovirus causes febrile pharyngitis (sore throat), acute hemorrhagic cystitis, pneumonia, conjunctivitis (pink eye).

Coxsackievirus (hand-foot-mouth disease): oval-shaped vesicles on palms and soles; vesicles and ulcers in oral mucosa.

155.

A male patient has fever and enanthesis.  As   a  result   of  the   examination  involving  serological  tests  he  has been  diagnosed  with fasciola hepatica. It was found  out  that  the  patient had  been infected  through  raw river  water.  Which stage  of fasciola  life cycle is invasive  for humans?

Explanation

           Clonorchis sinensis causes Clonorchiasis (asian liver fluke infection). Humans are infected by eating raw or undercooked fish containing the encysted larvae (metacercariae). After excystation in the duodenum, immature flukes enter (invade) the biliary ducts and differentiate into adults (adolescaria). The hermaphroditic adults produce eggs which are excreted in the faeces. Upon reaching fresh water, the eggs are ingested by snails which are the first intermediate hosts. The eggs hatch within the gut and differentiate first into larvae (rediae) and then into many free-swimming cercariae. Cercariae encyst under the scales of certain freshwater fish (second intermediate hosts) which are then eaten by humans.

156. A patient with high-titer antinuclear antibodies died from progressing  renal impairment. Autopsy  revealed   mesangioproliferative glomerulonephritis and a bacterial polypous  endocarditis. There was periarterial bulbar  sclerosis in spleen and  productive proliferative vasculitis  in skin. What is the most likely diagnosis?

Explanation

Systemic lupus erythematous (SLE, Libman-sacks disease) is the classic prototype of the multisystem disease of autoimmune origin, characterized by a bewildering array of autoantibodies, particularly antinuclear antibodies. It is characterized principally by injury to the skin, joints, kidney, and serosal membranes. Antinuclear antibody is directed against several nuclear antigens and can be grouped into 4 categories:

·        Antibodies to DNA

·        Antibodies to histones

·        Antibodies to nonhistone proteins bound to RNA

·        Antibodies to nuclear antigens.

SLE is a type III hypersensitivity reaction with formation of immune complexes. It can cause diffuse proliferative glomerulonephritis seen under the light microscope as “wire loop” of capillaries and granular under the immunofluorescence microscopy.

157.

Electronic microphotography of pulmonary alveoli’s wall  presents a  big cell. Its cytoplasm  has a lot of mitochondria, developed Golgi apparatus, osmiophil  lamellated corpuscles.  What  is the main function  of this cell?

Explanation

Alveolar epithelium is composed of type I and II alveolar cells and occasional brush cells. Type II alveolar cells or type II pneumocytes or septal cells are secretory cells. Their apical cytoplasm is filled with granules as stacks of parallel membrane lamellae, the lamellar bodies. They are rich in a mixture of phospholipids, neutral lipids and proteins that is secreted by exocytosis to form an alveolar lining, surface-active agent called surfactant.

158. A patient has food poisoning. Laboratory  analysis   revealed   a  culture of anaerobic gram-positive spore-forming bacteria. What  is the  most  likely kind  of the isolated  causative agent?

Explanation

          Some bacteria can form spores at the end of the stationary phase when nutrients are limited. Spores are highly resistant to heat and chemicals. Have dipicolinic acid in their core and they have no metabolic activity. Examples of species that form spores and the disease they cause:

              Species

                   Disease

    Bacillus anthracis

                   Anthrax

    Bacillus cereus

              Food poisoning

  Clostridium botulinum

                  Botulism

  Clostridium difficile

  Antibiotic-associated colitis

 Clostridium perfringens

Gas gangrene/Food poisoning

  Clostridium tetani

                   Tetanus

     Coxiella burnetii

                    Q fever

159. An  animal  has  an  increased tonus of   extensor   muscles.   This   the   result of    intensified    information   transmission   to   the   motoneurons  of  the   spinal cord through the following descending pathways:

Explanation

Vestibular nuclei receive impulses concerned with muscle tone and posture from vestibular apparatus and cerebellum. Vestibular nuclei in turn convey the impulses to different parts of the body through the anterior and lateral vestibulospinal tracts. Vestibulospinal tracts are concerned with adjustment of position of head and body during angular and linear acceleration; maintenance of muscle tone and posture; position of head and body during acceleration. Therefore, increase transmission of impulse through this tract leads to increase tone. Extensor muscles are also for balance (connected to vestibular apparatus). The inputs from the otolith organs project mainly to the lateral vestibular nucleus, which in turn sends axons in the lateral vestibulospinal tract to the spinal cord. The input from this tract exerts a powerful excitatory influence on the extensor (antigravity) muscles. When hair cells in the otolith organ are activated, signals reach the medial part of the ventral (anterior, motor) horn. By activating the ipsilateral (same side) pool of motor neurons innervating extensor muscles in the trunk and limbs, this pathway mediates balance and the maintenance of upright posture.

160.

A 70-year-old male patient died from acute coronary  insufficiency. He had knee joint  swelling, gonycampsis  and  gonalgia during his lifetime. Pathomorphologic examination of the deformed    joints and synovial membranes revealed membrane hyperaemia with multiple perivascular inflammatory infltrations made  by lymphocytes, plasmocytes and macrophagocytes. There was an accumulation of organized  fibrin covering some  areas  of synovium  membrane and looking  like rice grains in the articular liquid. What is the most likely diagnosis?

Explanation

        Rheumatoid (atrophic) arthritis is a chronic progressive inflammatory arthritis of unknown origin involving multiple joints and characterized by disorganization of connective tissue of the synovial membrane and articular cartilage and development of their deformation. Disorganization of the connective tissue cause increase in proteoglycans and glycosaminoglycans (GAGs) concentration in blood. Proteoglycans and GAGs are responsible for the physical properties of ground substance. Main morphological appearance of rheumatoid arthritis is synovitis. It has 3 stages: in the first stage, it is characterized by an acute inflammatory reaction with development of edema, hyperemia and infiltration of lymphocytes, plasma cells and macrophages. Small areas of superficial necrosis or superficial erosions are covered by fibrinoid deposits. Not infrequently 2-3mm “rice bodies” (rice grains) composed of fibrin, fibronectin, collagen and immunoglobulin are present in joint cavities of seropositive patients.

161. A  man  having  a  hearing  loss  after a head trauma was delivered to the neurosurgery department.  The  cause  of the  hearing  loss might  be the  damage  of the following lobe of cerebral cortex:

Explanation

IMG_9921        

 Temporal lobe of cerebral cortex includes 3 functional areas: primary auditory area, secondary auditory area and the area for equilibrium. The primary auditory area is concerned with perception of auditory impulses, analysis of pitch, determination of intensity and source of sound. Therefore, damage to this lobe of cerebral cortex can cause hearing loss.

162.

A  pregnant woman  was  registered in an antenatal clinic and underwent complex examination for a number of infections.  Blood  serum  contained IgM to  the  rubella  virus.  What  is this  result indicative of?

Explanation

 High IgM level usually indicate an acute or primary infection because, it is the first antibody to appear in response to initial exposure to an antigen. Secondary or reinfection show an increase in IgG. IgM antibodies appear early in the course of an infection, this makes it useful in the diagnosis of infectious diseases. Demonstrating IgM antibodies in a patient’s serum indicates recent infection or in a neonate’s serum, it indicates intrauterine infection.
163.

A   patient   with   tuberculosis   died from progressing cardiopulmonary decompensation. Autopsy in the region of the right lung apex revealed  a cavity 5 cm in  diameter  communicating with  lumen of  a  segmental   bronchus.  On  the  inside cavity walls are  covered  with caseous masses   with   epithelioid  and   Langhans cells beneath them.  What  morphological form of tuberculosis is it?

Explanation

Secondary tuberculosis usually results from reactivation of dormant, endogenous tubercle bacilli in a sensitized patient who has had previous contact with the tubercle bacillus. Reactivation typically begins in the apical or posterior segments (often 1st and 2nd segments) of one or both upper lobes (“simon’s foci”), where the organisms were seeded during the primary infection. There are 8 forms or stages of the secondary tuberculosis: Acute local tuberculosis; Fibrous local tuberculosis; Infiltrative tuberculosis; Tuberculoma; Caseous pneumonia; Acute cavernous tuberculosis; Fibrous cavernous tuberculosis; Cirrhotic tuberculosis.

Acute cavernous tuberculosis develops due to lyses of caseous necrosis and characterized by formation of the round cavity.

Fibrous cavernous tuberculosis is the most frequent form. Macroscopically, the lesions are spherical and cavitary (cavity can contain blood and blood clots); the so-called coin lesions. Microscopically, the outer wall of cavity shows fibrosis or sclerosis. Internal surface may be connected with bronchus. The wall of cavern has 3 membranes:

·        Internal membrane occurs by necrotic tissue

·        Middle membrane occurs by special granular tissue

·        External membrane occurs by connective fibrous tissue

      Tuberculoma consists of focus necrosis surrounded by fibrous capsule. Size of tuberculoma may be near 2-5cm.

164.

Vomiting  matters of a patient suspected  of  having  cholera   were  delivered   to  the  bacteriological  laboratory. The  material  was  used  for  preparing a \\\"hanging drop\\\" specimen. What type of microscopy will be applied for identification of the causative agent by its mobility?

Explanation

Vibrio cholera is the cause of cholera. It is curved, comma-shaped, gram negative rods. It is transmitted by fecal contamination of water and food, primarily from human sources. Clinically, watery diarrhea in large volumes is the hallmark of cholera. There are no red blood cells or white blood cells in the stool. Rice-water stool is the term often applied to the non-bloody effluent. Grows in an alkaline media. Dark-field and phase contrast microscopy have been used for screening fecal specimens for the presence of V. cholera. With these techniques, liquid stools are microscopically examined for the presence of organisms with typical darting (“shooting star”) mobility [analogous to ‘hanging drop’].

165. A patient with enteritis accompanied by  massive  diarrhea has  low  water  rate in the extracellular space, high water rate inside  the  cells and  low blood  osmolarity. What is such disturbance of water-electrolytic  metabolism called?

Explanation

        Hypoosmolar hypohydration (salt deficit) develops due to diarrhea, vomiting, sweating, adrenal insufficiency, hypokalemia, CNS lesions, salt-losing nephritis.

          Hyperosmolar hypohydration (water defcit) develops due to the loss of the fluid which lacks electrolytes e.g. in diabetes insipidus, hyperventilation.

          Isoosmolar hypohydration (isoosmotic loss): proportional volume decrease of fluids and electrolytes e.g. in blood loss, burns, diuretic therapy, especially diarrhea in cholera.

         Hypoosmolar hyperhydration (water excess) develops in acute renal failure (in the stage of anuria), Parhon’s syndrome (syndrome of inappropriate ADH secretion –SIADH).

         Hyperosmolar hyperhydration (salt excess) develops rarely e.g. in drinking salt (sea) water, infusion of hypertonic saline, adrenal hyperactivity or steroid therapy.

    Low water level extracellularly – Hypohydration

    Low blood osmolarity – Hypoosmolar

                Hypoosmolar hypohydration

166. Examination of a 6-month-old child revealed  a delay in closure  of the  occipital  fontanelle. When  should  it  normally close?

Explanation

font....

Fontanelles are large fibrous areas where several sutures meet; often called “soft spots” on an infant’s head. The two largest fontanelles are the anterior and posterior fontanelles on the superior surface of the neurocranium. The fontanelle in the back of the head (posterior fontanelle) usually closes by the time an infant is 2-3 months old. The fontanelle at the top of the head (anterior fontanelle) usually closes between 7-18months. In adults the remnants of the anterior fontanelle is the bregma and the posterior fontanelle is the lambda.

167.

A newborn child was found  to have reduced intensity of sucking, frequent vomiting, hypotonia. Urine and blood exhibit increased  concentration of citrulline. What metabolic process is disturbed?

Explanation

image       

In urea or ornithine cycle, the first two reactions leading to the synthesis of urea occur in the mitochondria, whereas the remaining cycle enzymes are located in the cytosol. The first reaction is formation of carbomoyl phosphate and the second reaction is formation of citrulline. Ornithine and citrulline are basic amino acids that participate in the urea cycle. The release of the high energy phosphate of carbomoyl phosphate as inorganic phosphate drives the reaction in the forward direction. The reaction product – Citrulline is transported to the cytosol where it condenses with aspartate to form argininosucinate. Increased concentration of citrulline in urine and blood indicates a defect in ornithine or urea cycle.

168. A  male  patient has been  diagnosed with  acute  radiation disease.  Laboratory examination revealed  a considerable reduction of platelet serotonin level. The likely cause of platelet serotonin reduction is the disturbed metabolism of the following substance:

Explanation

Serotonin, also called 5-hydroxytryptamine, is synthesized and stored at several sites in the body. It can be found in the intestinal mucosa, central nervous system and in platelets. Serotonin is synthesized from tryptophan, which is hydroxylated. The product, 5-hydroxytryptophan is decarboxylated to serotonin, which is also degraded by monoamine oxidase (MAO). Serotonin has multiple physiologic roles, including pain perception, affective disorders, and regulation of sleep, temperature and blood pressure.

169. In course of a conditional experiment the   development  of  mesenchyma  cells was  completely   inhibited.   Development of the following muscular tissue will be disturbed:

Explanation

Mesenchyme refers to loosely organized embryonic connective tissue regardless of origin. Undifferentiated embryonic mesenchymal cells are round/cuboidal in shape. During development, visceral myogenesis is shortly preceded by mesenchymal cell elongation. Undifferentiated embryonic mesenchymal cells from intestine (abundant visceral muscle), lung (some visceral muscle) or kidney (no visceral muscle); these cells differentiate into smooth muscle upon elongation.

170. A histologic specimen shows an organ\\\'s parenchyma which is presented by lymphoid  tissue  making  some  lymph nodes.   The   nodes   are   located   diffusively and contain a central artery. What anatomic formation might have such morphological structure?

Explanation

          The spleen is about the size of a clenched fist and is the largest lymphatic organ. It is located in the upper left quadrant of the abdominal cavity and has a rich blood supply. Most of the spleen consists of splenic pulp. Splenic pulp is divided into two regions: white and red pulp. White pulp consists of a thick accumulation of lymphocytes surrounding a central artery. Lymphocytes that aggregate around the central artery constitute the periarterial lymphatic sheath (PALS). The red pulp contains large numbers of RBCs that it filters and degrades.

171. A victim with a head  trauma in the temporal region has been  diagnosed  with epidural hematoma. What  artery  is most likely to be damaged?

Explanation

krushkrok No126 (2013) The maxillary artery gives 5 branches within the infratemporal fossa: deep auricular, anterior tympanic, inferior alveolar, accessory meningeal and middle meningeal/membranous arteries. The middle meningeal/membranous artery enters the cranial fossa through the foramen spinosum and gives branches to the dura mater of middle cranial fossa. It is the largest dural branch. The middle meningeal artery is most often damaged in fractures of the temporal bone producing epidural hematoma because it lies on the internal skull surface (pterion) which is the thinnest part of the lateral skull; therefore, it is vulnerable to fractures. The pterion is an area where the frontal, temporal, parietal and sphenoid (greater wing) articulate.
172.

A child has mental  and physical retardation, grave damage  of internal connective tissue.  Urine  analysis  reveals keratan sulfates.  What  metabolic process is disturbed?

Explanation

The Mucopolysaccharidoses are hereditary disorders that are clinically progressive. They are characterized by accumulation of glycosaminoglycans in various tissues, causing varied symptoms such as skeletal and extracellular matrix deformities and mental retardation. Mucopolysaccharidoses are caused by a deficiency of any one of the lysosomal hydrolases normally involved in the degradation of heparin sulfate and/or dermatan sulfate. This results in the presence of oligosaccharides in the urine because of incomplete lysosomal degradation of glycosaminoglycans.

173.

A   tooth    extraction  in   a   patient with chronic persistent hepatitis was complicated with prolonged hemorrhage. What  is the  reason  for the  haemorrhagic syndrome?

Explanation

           Liver disease (hepatitis) may result in a number of defects in hemostasis. Except vitamin K deficiency, they may lead to the reduced synthesis of factors of coagulation. The reduced synthesis of coagulation factors may be the result of severe hepatocellular damage. There are 3 main stages in the blood coagulation cascade:

·        Formation of the thromboplastin complex

·        Transformation of prothrombin to thrombin

·        Transformation of fibrinogen to fibrin

In hepatitis, there will be reduced synthesis of thrombin and fibrin; but thrombin comes before fibrin in the coagulation cascade, therefore, decrease in thrombin production is a more correct answer in this case.

174. Sex chromosomes of a woman  didn’t separate and  move  to the  opposite  poles of a cell during  gametogenesis (meiosis). The ovum was impregnated with a normal spermatozoon. Which chromosomal disease can be found in her child?

Explanation

image

In meiosis, two members of a pair of homologous chromosomes normally separate during the first meiotic division, so that each daughter cell receives one member of each pair. Sometimes, however, separation does not occur (nondisjunction) and both members of a pair move into one cell. As a result of nondisjunction of the chromosomes, one cell receives 24 chromosomes and the other receives 22 instead of the normal 23. When at fertilization, a gamete having 23 chromosomes (spermatozoon) fuses with a gamete having 24 or 22 chromosomes, the result is an individual with either 47 chromosomes (47 XXX – trisomy X) or 45 chromosomes (45 XO – monosomy X, Turner’s syndrome). In women, the incidence of chromosomal abnormalities including nondisjunction, increases with age especially at 35years and older.

krushkrok No105 (2014)

175. A  female  patient was administered loratadine for allergic cheilitis treatment. What is the mechanism  of the drug’s action?

Explanation

         Loratadine is an antihistamine (antiallergic drug) which refers to the classic H1-receptor blocker. These compounds do not influence the formation or release of histamine. Rather, they block the receptor-mediated response of a target tissue. They are much more effective in preventing symptoms than reversing them once they have occurred. These agents, desloratadine, fexofenadine, and loratadine show the least sedation i.e. least effect on CNS (they don’t induce sleep).

176.

Ultramicroscopical examination of \\\"dark\\\" hepatocyte  population in  the  cell cytoplasm  detected a developed granular endoplasmic reticulum. What function has this organella in these cells?

Explanation

        Granular or rough endoplasmic reticulum (i.e. rough surface) is a region of endoplasmic reticulum associated with ribosomes. It is the site of protein synthesis and modification of newly synthesized proteins.

177.

A doctor  prescribed a cephalosporin antibiotic to the patient after appendectomy for  infection   prevention.  Antimicrobial activity of this group of antibiotics  is based upon   the   disturbance  of  the   following process:

Explanation

     The cephalosporins are β-lactam antibiotics that are closely related both structurally and functionally to the penicillins. Cephalosporins like the penicillns, inhibit bacterial cell wall synthesis and are considered bactericidal against susceptible organisms.

         Penicillins inhibit the synthesis of bacterial cell walls and are considered bactericidal too. They bind penicillin binding proteins (transpeptidases) and block transpeptidase cross-linking of peptidoglycan in cell wall. They also activate autolytic enzymes.

178.

A patient underwent an extraction of a part of a CNS structures by medical indications.  As a result  of the  extraction the patient developed atony,  astasia,  intention tremor, ataxy and adiadochokinesis. Which  part  of  CNS  structure had  been extracted?

Explanation

During cerebellar lesions, there  are disturbances in posture, equilibrium and movements. Disturbances in movements: Speech disorders, ataxia, asynergia, asthenia (weak muscle contractions with characteristically rapid onset of muscle fatigue), Dysmetria, Intention tremor, astasia (loss of the capacity for sustained tetanic contractions), nystagmus, rebound phenomenon, dysarthria, adiadochokinesis, atonia (lack or impairment of muscle tone).

NB: Scanned or staccato speech is as a result of cerebellar disorders; while monotonus speech is as a result of parkinson's disease.

179.

A  patient has  been  diagnosed  with influenza.  His  condition  became   drastically worse after taking antipyretic drugs. His consciousness  is confused, AP is 80/50 mm  Hg,  Ps  is 140/m, body  temperature dropped down to 35, 8oC . What complication developed in this patient?

Explanation

His condition became worse after taking antipyretic drug. Antipyretic drugs block prostaglandin synthesis by inhibiting cyclooxygenase (COX) enzyme at the thermoregulating centers in the hypothalamus and at peripheral target sites. In case of toxicity of this drug, it can result in coma followed by cardiovascular collapse and respiratory arrest due to its CNS depressant activity. Consciousness is confused; arterial blood pressure is low and pulse rate is very high – these are clear signs of collapse.

180.

A  patient consulted  a doctor  about loss of taste sensitivity on the tongue  root. The  doctor  revealed  that  it is caused  by nerve affection. Which nerve is it?

Explanation

        The glossopharyngeal nerve (CN IX) is the mixed type nerve. It comprises the motor, sensory and autonomic fibers. It gives numerous branches but the lingual branches run to the posterior third of lingual mucosa. The nerve comprises the fibers of general sensitivity and the gustatory (taste) fibers that supply the taste buds.

·        Anterior 2/3 of tongue is supplied by chorda tympani, a branch of CN VII (facial nerve) for taste sensation.

·        Anterior 2/3 of tongue is supplied by CN V3 (mandibular division of the trigeminal nerve) for tactile sensation.

·        Posterior 1/3 of tongue is supplied by CN IX (glossopharyngeal nerve) for taste and tactile sensation.

181.

Which muscle contraction will be observed  in  the   upper   extremity  during holding  (but  not  moving)  a load  in a certain position?

Explanation

             Muscle contraction is classified into 2 types: Isotonic and Isometric.

·        Isotonic contraction: the tension in the muscle remains the same but the length of the muscle fiber is changing (Iso = same; tonic = tension) e.g. simple flexion of arm, where shortening of muscle fibers occurs but the tension does not change.

·        Isometric contraction: the length of muscle fibers remains the same but the tension is increased e.g. pulling any heavy object when the muscles become stiff and strained with increased tension but the length does not change. Holding (but not moving) – length does not change.

182.

An older  woman  has been  hospitalised for acute pain and edema  of the right hip joint that appeared after a fall. Objectively: the  hip is adduced inwards,  hip joint movements are  impaired. The  patient is most  likely  to  have  a  fracture of  the following bone or bone part:

Explanation

          The femur consists of a shaft (body) and two ends, superior/proximal and inferior/distal. The neck of the femur is trapezoidal, with its narrow end supporting the head and its broader base being continuous with the shaft. Fractures of the femoral neck is a very common occurrence in older people as a result of a slight stumble if the neck has been weakened by osteoporosis.

183.

A   45-year-old   woman   has   breast cancer. Her left arm has symptoms of lymphatic  system insufficiency - limb edema,  lymph  node  enlargement. What form of lymphatic circulation  insufficiency is it?

Explanation

          Types of lymph-circulatory insufficiency:

·        Mechanical insufficiency

·        Dynamic insufficiency

·        Insufficiency of absorption/Resorption insufficiency

Mechanical insufficiency is present where lymph flow is hindered by a mechanical factor. The mechanical factor may be of an anatomical, organic nature e.g. occlusion of lymphatics by obstructive lymphangitis, thrombosis of the lymphatics (e.g. during cancer metastasis), filariasis.

The other two types has to do with a disbalance of proteins that upsets the filtration and absorption pressures.

184.

Study of conversion  of a food colouring agent  revealed  that  neutralization of this  xenobiotic   takes  place  only  in  one phase  - microsomal   oxydation.   Name  a component of this phase:

Explanation

     The P450 system is impotant for the metabolism of many endogenous compounds (such as steroids, lipids etc) and for the biotransformation of exogenous substances (xenobiotics). Cytochrome P450, designated as CYP, is a superfamily of heme-containing isozymes that are located in most cells but are primarily found in the liver and GIT. Phase I reactions utilizing the P450 system (also called microsomal mixed function oxidases). The oxidation proceeds by the xenobiotic binding to the oxidized form of cytochrome P450 and then O2 is introduced through a reductive step, coupled to NADPH: cytochrome P450 oxidoreductase.

185. While  performing an inguinal  canal operation on account  of hernia  a surgeon damaged the canal\'s contents. What exactly was damaged?

Explanation

\"image\"            

 What forms the walls of the ingunal canal:

·        Anteriorly: the aponeuroses of the external and internal oblique muscles.

·        Posteriorly: the transversalis fascia.

·        Superiorly: the arching fibers of the internal oblique and transverses abdominis muscles.

·        Inferiorly: the inguinal ligament.

Contents:

·        Men: Spermatic cord (funiculus spermaticus) and the ilioinguinal nerve

·        Women: round ligament of the uterus (ligamentum teres uteri) and the ilioinguinal nerve.

      Since the gender was not specified in the question, then it means the question s referring to inguinal canal content in men, which is the Funiculus spermaticus.

186.

A  patient with  myocardial  infarction was admitted to the cardiological department. For  pain  relief  it  was  decided   to   potentiate  fentanyl   action   with  a  neuroleptic.  Which  of  the  following  neuroleptics is the  most  suitable  for neuroleptanalgesia?

Explanation

          Droperidol is often combined with a potent narcotic analgesic such as fentanyl to produce neuroleptanalgesia. It is used for neuroleptanalgesia because of its quick effect (action), short duration of action, myorelaxant and antihypertensive effects. Droperidol has quick and short effects, produces hypotension and myorelaxantia.

         Fentanyl + Droperidol = Talomonal (neuroleptanalgesia)

187.

A 25-year-old man has spent a long time in the sun under  high air humidity.  As a result of it his body temperature rose up to 39oC . What pathological process is it?

Explanation

       Hyperthermia (i.e. marked warming of core temperature) is not mediated by pyrogens and there is no resetting of the hypothalamic set point. It may be accidental or therapeutic.

188. A  26-year-old  man  is in the  torpid shock phase  as a result  of a car accident. In blood:  3, 2 · 109/l. What  is the  leading mechanism  of leukopenia development?

Explanation

Shock is a form of stress which is accompanied by the release of stress hormones (catecholamines, cortisol). Stress-induced changes in blood leukocyte distribution may represent an adaptive response. This represents a redistribution of leukocytes from the blood to other organs such as the skin, draining sentinel lymph nodes and other compartments. Such leukocyte redistribution may enhance immune function in compartments to which immune cells traffic during stress.

189.

A patient consulted a doctor  about  a sensation of  imbalance  which  appeared after a trauma. Which nerve is damaged?

Explanation

image        

image

The receptor areas of the vestibular labyrinth are represented with the following structures:

·        Macula of Utricle

·        Macula of Saccule

·        Ampullary crests

The first two detect linear motion, while the ampullary crest detect angular motion. The vestibulocochlear nerve supply these receptors. Both maculae consist of the sensory hair cells covered with jelly-like susbstance. The substance contains the crystals of calcium carbonate called otoliths (vestibular otolith). The utricle and saccule detect linear movement, also contributing to balance. The ampullary crests reside within each membranous ampulla. They also comprise the sensory hair cells covered with the same jelly-like substance called the ampullary cupula. The ampullae is the sensory organ in the semicircular canal that sense angular (rotational) acceleration of the head, thereby regulating balance. NB: carousel is a revolving belt.

190. An alcoholic has alcoholic psychosis with evident psychomotor agitation. What neuroleptic drug should be administered  for emergency  care?

Explanation

·        Aminazine – neuroleptic (it is better than reserpine for emergency care. It has also proved to be useful in control of intracted hiccup).

·        Diazepam – tranquilizer

·        Sodium bromide – sedative

·        Reserpine – neuroleptic

·        Halothane – general anesthesia

191.

A  patient who  has  been  treated  in a  neural   clinic  and  has  been   taking   a sedative  for  a long  time  got  the  following complication: cough, rhinitis, epiphora. What drug caused these disturbances?

Explanation

        Doses of bromides have to be determined individually. The bromides are excreted from the organism rather slowly (50-60days), so they may be accumulated and cause the signs of chronic poisoning (bromism). The irritative action of bromides leads to inflammation of the mucous membranes which is accompanied by cough, rhinitis, conjunctivitis and diarrhea.

192. The liver puncture biopsy of a patient with hepatocellular insufficiency revealed hydropic  and  ballooning degeneration of hepatocytes, necrosis of certain  cells, presence of  Kaunsilmen’s bodies.  Portal and lobular stroma were infiltrated mostly with    lymphocytes    and     macrophages as well as with a small number of polymorphonuclear lymphocytes.  What is the most likely diagnosis?

Explanation

        The morphologic changes in acute viral hepatitis are virtually the same regardless of the causative agent and can be mimicked by drug reactions. Grossly, the liver is slightly enlarged. Histologically, the major finding is hepatocellular injury characterized by necrosis of scattered hepatocytes; isolated liver cells or small cell clusters appear as eosinophilic rounded-up cells (apoptotic bodies, councilman’s bodies). Degenerated hepatocytes may also appear ballooned. Macrophages may phagocytize the necrotic hepatocytes and may accumulate clumps of lymphocytes and macrophages. Confluent necrosis may lead to bridging necrosis connecting portal-to-portal, central-to-central or portal-to-central regions of adjacent lobules, signifying a more severe form of acute hepatitis.

193. In  order  to  accelerate healing  of  a radiation ulcer  a vitamin  drug was administered. What drug is it?

Explanation

 Several studies, such as the one published in the June 2008 edition of the “Journal of vascular nursing”, have shown reduced levels of vitamin A in patients suffering from leg ulcers, indicating a connection between the two, probably because vitamin A helps to form and maintain healthy skin, mucous membrane and teeth. Vitamin A and the amino acid glutamine help to regenerate healthy epithelial cells. Therefore, it can be used to treat radiation ulcer.
194.

After  inoculation of the material obtained from  the  pharynx  of an angina patient onto the blood-tellurite agar, grey colonies  could  be  observed.   They  were  4-5 mm in diameter, radially  striated (in form  of  rosettes). Microscopical   examination  revealed  gram-positive bacilli  with clavate  swollen ends  arranged in form of wide-spread fingers. Identify  these  microorganisms:

 

Explanation

Diphtheria bacteria (Corynebacterium diphtheria) is Gram positive, pleomorphic, often club-shaped rods and are arranged in palisades or in V (at an angle) or L-shaped formations. Media used for isolation are Tellurite agar & Lὄffler medium. Lὄffler nutrient medium consists of coagulated serum & nutrient broth. Selective indicator medium containing tellurite are used in selective culturing. K tellurite is used to inhibit the accompanying flora.

195.

Cytogenetic examination of a patient with reproductive dysfunction  revealed normal karyotype 46 ХY in some cells, but most cells have karyotype of Klinefelter’s syndrome  - 47 ХХY. Such cell heterogeneity is called:

Explanation

FullSizeRender (1)

krushkrok No70 (2007)

Barr body is an inactive X-chromosome. So a boy (XY) with an inactive X-chromosome must have an additional X-chromosome – XXY (Klinefelter’s syndrome). Causes :

* nondisjunction (maternal and paternal nondisjunction in meiosis I)

* Mosaicism: with the karyotype being 46, XY/47, XXY

Manifestations: gynecomastia, female pattern of pubic hair distribution, no facial hair, high voice.

196.

A  patient  presents with  acne  and inflammatory alterations  of  facial  skin. Microscopical   investigation of  lesion  foci has  revealed   live  arthropods  sized  0,2-0,5  mm.  They  have  prolate  vermiform form and four pairs of thin short limbs located  in  the  middle  part  of  the  body. The revealed  arthropods cause:

Explanation

krushkrok No98 (2010)          

Demodex folliculorum and Demodex brevis: they are species of face mite. When large numbers of D. folliculorum are found in humans, the infestation is known as Demodicosis. D. folliculorum is semi-transparent elongated organism consisting of a head, neck, body and tail. As an adult, it can measure 0.1-0.4mm in length and possess 4 pairs of short legs near its head and neck region. In the course of time, because of chronic progradient course, pathologic process results in formation of inflammatory nodes, infiltrates and persistent vascular dilatation. Hyperemia of facial skin and eruptions are also seen.

197.

Surgical   approach  to   the   thyroid gland     from     the     transverse    (collar) approach involves opening of interaponeurotic suprasternal    space.     What     anatomic structure   localized    in    this    space    is dangerous to be damaged?

Explanation

The anterior jugular vein arises from small superficial veins of the sublingual area. The veins descend to the manubrium of sternum and merge to form the jugular venous arch. The lateral ends of the arch open into the external jugular vein before it joins the venous angle.

NB: veins are located superficially to arteries.

198.

An  injured  person  was delivered  to the  hospital  with a penetrating wound  in the  left lateral  region  of abdomen. What part  of the  large  intestine is most  likely damaged?

Explanation

imageimage

Anterolateral abdominal wall has 9 regions and 4 quadrants (RUQ, LUQ, RLQ, LLQ)

·        Right lateral abdominal region: Ascending colon (colon ascendens), right kidney, right ureter and loops of small intestine.

·        Umbilical region: Transverse colon (colon transversum), head of pancreas, duodenum (except superior part)

·        Left lateral abdominal region: Descending colon (colon descendens), left kidney, left ureter and loops of small intestine.

·        Left inguinal region: Sigmoid colon (colon sigmoideum), left ureter, left external iliac artery of artery and vein.

·        Right Inguinal region: Caecum, vermiform appendix, right ureter

199.

A   4-year-old   child   presents  with general weakness, sore throat and deglutitive  problem.  After   his  examination a doctor  suspected  diphtheria and sent  the material to the bacteriological laboratory. In order  to determine the diphtheria causative   agent   the  material should   be inoculated into  the  following  differential diagnostic medium:

Explanation

Diphtheria bacteria (Corynebacterium diphtheria) is Gram positive, pleomorphic, often club-shaped rods and are arranged in palisades or in V (at an angle) or L-shaped formations. Media used for isolation are Tellurite agar & Lὄffler medium. Lὄffler nutrient medium consists of coagulated serum & nutrient broth. Selective indicator medium containing tellurite are used in selective culturing. K tellurite is used to inhibit the accompanying flora.

200.

Examination of the anterior abdominal  wall  of  a  pregnant woman  revealed a  tumour-like  formation  that   arose   on the  spot  of a tumour that  was removed two years ago. The neoplasm  was well-defined,  dense, 2х1 cm large. Histological examination revealed that the tumour was composed  of differentiated connective tissue with prevailing  collagen fibres. What tumour might be suspected?

Explanation

         Desmoid fibroma is a benign connective tissue tumor. It a kind of dense fibroma and characterized by infiltrating growth and relapses. It is composed of banal, “tame-loking” fibroblasts that do not metastasize. More often it is located on the anterior abdominal wall.